Categories
AEA Berkeley Chicago Cornell Economist Market Economists Johns Hopkins M.I.T. Princeton Stanford Yale

M.I.T. Memo regarding potential hires to interview at AEA Dec meeting, 1965

 

This artifact provides us a glimpse into the demand side of the market for assistant professors of economics in the United States as seen from one of the mid-1960’s peak departments. The chairperson of the M.I.T. economics department at the time, E. Cary Brown, apparently conducted a quick survey of fellow department heads and packed his results into a memo for his colleagues who in one capacity or the other would be attending the annual meeting of the American Economic Association held in New York City in the days following the Christmas holidays of December 1965. The absence of Harvard names in the memo probably only indicates that Brown presumed his colleagues were well aware of any potential candidates coming from farther up the Charles River.

From Brown’s memo, Duncan Foley (Yale) and Miguel Sidrauski (Chicago) ended up on the M.I.T. faculty as assistant professors for the 1966-67 academic year. John Williamson was a visiting assistant professor that year too.

_____________________________

Dating the Memo

The folder label in the M.I.T. archives incorrectly gives the date Dec. 28-30, 1969, where the 1969 has been added in pencil.

Two keys for dating the memo.  Brown’s comment to John Williamson (York): “Wants a semester here, Jan.-June 1967″.  “Solow is hearing paper at meetings” (Conlisk of Stanford) who presented in the invited doctoral dissertation session “The Analysis and Testing of the Asymptotic Behavior of Aggregate Growth Models” (affiliation given as Rice University (Ph.D., Stanford University) where Solow was listed as a discussant. AEA’s 78th Annual Meeting was held in New York City at the end of December 1965.

_____________________________

Memo from E. Cary Brown to M.I.T. faculty going to Dec. 1965 AEA meeting

[Pencil note: “Put in beginning of 1966-7”]

Memorandum Regarding Personnel Interviews in New York

To: Department Members Attending AEA Convention
From: E. C. Brown

University of Chicago

Sidrauski, Miguel (26). International Trade, Monetary Theory, Economic Growth, Mathematical Economics

Thesis—“Studies in the Theory of Growth and Inflation” under Uzawa
References: Harberger, Johnson, Lewis

[He came here a year ago to ask about a short-term appointment before he returned to Argentina. Griliches believes him to be tops. Had him in class myself and he was first rate. Called him on phone last week and he still wants to be had.]

 

Thornber, Edgar H. (24). [H. = Hodson] Econometrics, Mathematical Methods, Computers

Thesis—“A Distributed Lag Model: Bayes vs. Sampling Theory Analyses” under Telser
References: Griliches, Zellner

[Supposed to be equal of Sidrauski. Heavily computer oriented. Doesn’t sound interesting for us, but we should talk to him.]

 

Treadway, Arthur. Mathematical Economist

Thesis on the investment function

[A younger man who, according to Svi [sic], regards himself as the equal of the above. Stronger in mathematics, and very high grades. Wasn’t on market because thesis didn’t appear as completable. Now it seems that it will be and he wants consideration.]

 

Evenson, Robert E. (31). Agricultural Economics and Economic Growth, Public Finance

Thesis—“Contribution of Agricultural Experiment Station Research to Agricultural Production” under Schultz
References: Gale Johnson, Berg

[He is just slightly below the others. Mature and very solid and combines agriculture and economic growth where we need strength.]

 

Gould, John (26).

(Ph.D. in Business School)

[Bud Fackler mentioned him as their best. Uzawa and Griliches are trying to get the Econ. Dept. to hire him. Franco knows him and is after him.]

 

Princeton

Klevorick, Alvin (22). Mathematical Economics, Econometrics, Economic Theory

Thesis: “Mathematical Programming and the Problem of Capital Budgeting under Uncertainty” (Quandt)
References: Baumol, Kuhn

[Apparently the best they have had for some time. Young and very brash.]

 

Monsma, George N. (24). Labor Economics, Economics of Medical Care, Public Finance

Thesis: Supply and Demand for Medical Personnel” (Harbison)
References: Patterson, Machlup

[Dick Lester was high on him. While not a traditional labor economist, he works that field.]

Silber, William L. (23). Monetary Economics, Public Finance, Econometrics

Thesis: “Structure of Interest Rates” (Chandler)
References: Goldfeld, Musgrave, Quandt

[One of their best four. Not sure he sounds like what we want in fields, however.]

 

Grabowski, Henry G. (25). Research and Development, Econometrics, Mathematical Economics

Thesis: “Determinants and Profitability of Industrial Research and Development” (Quandt)
References: Morgenstern, Baumol

[Lester says he is good all around man. His field makes him especially interesting.]

 

Stanford

Conlisk [John]— Economic growth and development

[Arrow has written about him, recommending him highly. His field should be interesting. Solow is hearing paper at meetings.]

 

Bradford [David Frantz]— Public finance

[Has been interviewed up here, but more should see him who wish to.]

 

Yale

Foley [Duncan Karl] (Probably not at meetings. Best Tobin’s had.]

Bryant [Ralph Clement] (Now at Federal Reserve Board. Number 2 for Tobin]

 

York

Williamson, John

[Wants a semester here, Jan.-June 1967. Alan Peacock at meetings.]

 

Johns Hopkins

[Ask Bill Oakland]

 

University of California, Berkeley

[Ask Aaron Gordon or Tibor Scitovsky.]

 

Cornell

Bridge [John L.] — Econometrics, Foreign Trade

Lindert [Peter]— International Economics

[Their two best as indicated in their letter to Department Chairman.]

 

Buffalo

Mathis, E.J. [Ask Mitch Horwitz if it’s worth pursuing.]

 

Columbia U.

[Ask Bill Vickrey]

 

Pittsburgh

Miller, Norman C. (26). International Economics; Money, Macro, Micro and Math Economics

Thesis: “Capital Flows and International Trade Theory” (Whitman)
References: Marina Whitman, Jacob Cohen, Peter Kenen, Graeme Dorrance

[Letter to Evsey Domar from Mark Perlman (Chm.) recommending him to us for further training.]

 

Source: Massachusetts Institute of Technology. Institute Archives and Special Collections. MIT Department of Economics records, Box 1, Folder “AEA Chairmen MEETING—Dec. 28-30, 1969 (sic)”.

Image Sources:  Duncan Foley (left) from his home page. Miguel Sidrauski (right) from the History of Economic Thought website.

Categories
Funny Business M.I.T.

MIT. Faculty Christmas Party Skit. Seven Stages of a Student, 1964

 

The following faculty skit from the M.I.T. economics department apparently had multiple authors. The last act was penned by Robert Solow–it was the only part of the script that was written in long-hand and only Act VI of this skit is found in Robert Solow’s papers in the Duke archives). Unfortunately Act V “The Thesis Defense” was not included in the Graduate Economics Association (1961-67) folder of the Economics Department Records at the M.I.T. Archives.

Attempts at racial, ethnic and gendered humor need no further comment than to note their respective shelf-lives expired two generations ago.

____________________________

GEA Christmas party 1964

Appetite of a Man; Income of a Boy
(The Seven Stages of a Student)
a play in six acts

Cast

Student—played by [blank]
Registration Officer—played by [blank]
Other students, professors, deans, etc.

Act I—The Admission Interview
Act II—Registration
Act III—Talk to the First-Year Class
Act IV—The General Examination
Act V—The Thesis Defense
Act VI—Employment: Going out into The World

TO THE CAST: IF YOU DON’T LIKE A LINE, IMPROVE ON IT.

 

Act I: The Admission Interview—Student and Admissions Committee

Student Applicant: Sir, I believe you have an economics department here at MIT. Can you tell me why?

Prof. 1: Why does a dog have fleas? To keep things stirred up. But how did you hear about it?

Student: Oh, I follow the basketball scores very closely. If this is the Admissions Committee, I’d like to apply.

Prof. 2: How did you do in college?

Student: I averaged 27 points a game.

Prof. 3: No, we want to know how you did in your college work. Tell us something about your grades, about your preparation, especially in economics and mathematics.

Student: We’ll get to that jazz in due course. But let me remind you, I am interviewing you, not you me. You tell me about fellowships, about student loans, and about parking stickers, how are the students fixed for the things that count.

Prof. 2: Well, you can get a Woodrow Wilson.

Student: If I was going to deal with Woodrow Wilson, I’d have gone to Princeton where they have the school and $35 million, to say nothing of $5.3 million on the side in history.

Prof. 1: There’s the National Science Foundation.

Student: Whose got the balance-of-payments disequilibrium. I am talking of how much money you are going to give me, not how much money I am going to bring to you. Now get this straight: I have expenses. These Triumphs cost money to maintain, and my girl likes steak. I also want refinance my stock market operations from my broker’s 6% to what I understand are your 2% loans for students. You give me tuition plus $5,000, plus another $5,000 loan, plus a ticket to park my car inside the Grover C. Hermann Building, or I’m on my way to Yale on a NASA.

Chorus: Nasa’s in the cole, cole groun. [Song by Stephen Foster “Masa’s in the Cold, Cold Ground”]

 

Act II—Registration

Reg. Off.—This stuff is pretty cut and dried: 14.121 Bishop, if you’re strong enough to turn the crank and carry the script; 14.451, mathematics, statistics, and a course like history, labor, trade, money.

Student: Whoa, back. Not so fast. First, let’s worry about the languages. There’s Spanish.

Reg. Off. We don’t let students take Spanish unless they are interested in development in Latin America, and have a need to read the limited literature.

Student: I guess I prefer Portugese.

Reg. Off. Development in Brazil.

Student: The Bossa Nova. But after the language, I think I’ll start on the minor: some of the 15 courses: Social Distance and Proximity during and After the Office Party, that sounds interesting; and maybe Design Packaging, how to get a nickel’s worth of stuff into a buck’s package; and Engineering Social Change for Chemical Engineers, or what to do after the Stink Bomb drops by mistake.

Reg. Off. And 14.121

Student: and some courses in the soft option: what is it this year, trade, labor, development? What about that course I heard about in which the students all graded each other on how they related to one another—a children’s party with an A for each kid.

Reg. Off. And 14.121.

Student: And a course at Harvard with real razz-matazz: Lady Jackson [Barbara Mary Ward, Baroness Jackson of Lodsworth, a development economist], and Man Galbraith, and Senor Chenery, and Don [here the honorific title for a nobleman] Hirschman.

Reg. Off. Look pal. Everybody takes 14.121.

Student: You can’t mean that we do too, those of us here on athletic scholarships.

 

Act III—Reg. Off. To the First-Year Class.

Student 1 whispering to Student 2: They say it’s a terrible experience. Students faint and dragged out. Chills come over them. There’s a lot of talk of Cs and Ds, and fellowships being taken away, and students walking the plank.

Student 2, whispering to Student 1: Naw, it’s no worse than a bad cold, and you’re not a man until you’ve had it.

Reg. Off. “Look to the right of you, look to the left of you. Of the three of you, only one will be here next term.” What famous book on economics started that way and the edition had to be suppressed. You students really have it made. Appetite of a man; income of a Boy. How much better you are off than my older colleagues, with their income of a man, and appetites of a boy.

Student: What about Grades?

Reg. Off. Grades? Grades? Who pays any attention to grades? Grades are trivial; the second order of smalls; a mere epsilon, nothing. Of course you need one A to get tuition money for the second year, and a second A for every $100 of coffee-and-cakes money. But grades? Who needs ‘em? They’re for undergraduates, for grade hounds, for Phi Beta Kappa or College-Bowl kids. Concentrate on higher things like saying Stolper-Samuelson and not (repeat not) Samuelson-Stolper.

 

Act IV: The General Examination

Prof 1: Good morning, Mr. Mittlablook.

Prof 2: Good morning, Mr. Pswoom.

Prof 3: Good morning, Mr. Pixyquicksel

Student (aside): Isn’t it lovely, they all know my name after two years.

Prof 1: Let’s get down to business.

Student: Must we?

Prof 2: What would you like to be examined in first? I see we have economic theory, economic history, and textbook writing and consulting fees.

Student: I am afraid I am not responsible for any of those.

Prof 3: We would all like to say the same.

Student: I was told when I came that I could be examined in comparative economic systems, the difference between capitalist and socialist economies, and free enterprise sink or swim.

Prof 1: Those fields were discontinued this morning.

Prof 2: Yes, I am afraid you’ll have to take the exam in economic theory and history.

Student: I think that is dreadfully unfair.

Prof 3: Well let me start you off by asking you a question in economic history. Consider the period which used to be known as the industrial revolution. This was accompanied, as you know by a large population explosion. Would you discuss the relative roles of (a) men and (b) women, in this development?

Student: Well, I suppose you could say that they each contributed something but the truth lies somewhere in between.

Prof 1: Wrong; you are supposed to say that the roles are neither reflexive, symmetric, nor transitive.

(STAGE DIRECTION: The last time we tried that line we stepped on it. It should be read with greater expression.)

Prof 2: That question was meant to combine economic history and economic theory. Let me ask you one about the history of economic theory. Name a business cycle theorist who was also a Russian cowboy.

Student: Evsey Domar.

Prof 3: Wrong again; Tugan Baranowsky. (general groans)

Prof 1: Now we come to your third field which is, I understand, professor imitating.

Student: Yes, I have learned to make noises like a professor now and then.

Prof 2: That will be no doubt fascinating at the Christmas Party.

Prof 3: Imitate a professor.

Student: How can I imitate a professor when I am a professor imitating a student?

Prof 1: Imitate a professor imitating a student imitating a professor.

Student: I am not responsible for infinite sequences.

Prof 2: Could you leave the room while we discuss you please. You’ll hear from us in about three years Thursday. (student leaves)

Prof 3: Well, what shall we do? He is a bright boy but he didn’t do too well.

Prof 1: On the other hand, I thought he was a stupid boy but did very well.

Prof 2: I see that as usual we are in complete agreement.

Prof 3: There is only one thing we can do. Give him an excellent plus and tell him not to write his thesis.

END OF SCENE.

 

Act V. The Thesis Defense
[missing]

 

Act VI. Employment

[Handwritten mimeo, author: Robert Solow]

Student sitting grandly in chair, feet on table, cigar? Del Tapley shows in two interviewers, I1 and I2.

D.T.: Mr. Auster, sir, these servile wretches represent Princeton and the University of Minnesota. They have an audience, I mean appointment, with you.

  1. Come in chaps. Sorry to have to see you two at the same time like this, but my schedule is very crowded. I have to squeeze in the rest of the Big Ten this morning; and this afternoon I’m seeing Yale, Chicago, and a representative of the Free Speech Movement at Berkeley.

I1: You mean…

A: Yes. Radner almost made it with that beard. But somehow he was just a little too much Commander Whitehead [president of Schweppes U.S.A. and featured in the Schweppes advertisements] and not enough Fidel. Anyhow, he’s been dropped. The FSM [Free Speech Movement] has eliminated the middleman. Mario [Savio, a leader of the Free Speech Movement] may come himself. We’re sending a delegation to meet him, at the B&A [Boston and Albany Railroad] yards. Must remind Marcelle and Cynthia not to comb their hair. But what can I do for you, or vice versa?

I2: Well, we do feel Minnesota has a lot to offer a young man…

A: Stop feeling and start offering.

I2: Sorry, sir. Our special CRAP salaries…

A: What?

I2: Charles River Assistant Professorships—they start at $17,500. Unfortunately since Walter [Walter Heller] got back they’re only allowed to go up at 3.2% a year, but we try to make it up in sly ways. That’s for 9 months, of course…

A: Nine months?

I2: Well, not nine full months—we do have a special slush fund to cover the week between terms. And we send you all expenses paid to the annual Christmas meeting any time it is in Miami. Of course if it’s not in Miami, we just send you to Miami.

A: Only fair. Pretty cold out there. Of course Adelman goes to the Virgin Islands every winter.

I1: I’ve heard that Solow curls up in a hollow tree in Concord and hibernates.

A: How can they tell? Never mind. Seventeen-five sounds reasonable. What about the teaching load?

I2: Teaching load? I didn’t realize you were actually willing to do any teaching. In that case you begin at 20,000, naturally. What were you thinking of teaching?

A: Why near-decomposability, of course. Is there anything else? By the way, do you have a Community Antenna Television Association [CATV]?

I2: No, but…

A: No buts. I’m not interested. But you ought to see Bridger Mitchell [MIT graduate student, a telecommunications expert with Charles River Associates] while you’re here—I understand he won’t go to any university within 100 miles of a CATV. Tell me about Princeton.

I2: But I haven’t told you about the 13/9th summer pay, or the every-other-year sabbatical, or how you get Leo Hurwicz for a research assistant, and girls, girls, girls,…

A: Sorry. Not interested. Actually, I’m not anxious to leave the East coast anyway. To tell you the truth, I’m not even sure how to do it. Tell me about Princeton.

I1: I do hope you will think seriously about Princeton, sir. We’re rather different from this Johnny-come-lately place, you know. More like a way of life. Gentlemen-scholars. Culture. Charm[?] Ivy. Yet intellect. We did have Einstone, you know.

A: You mean Einstein?

I1: Well, we suggested he change his name. Don’t think we’re stuffy, however Princeton had a Negro student as long as 30 years ago. And one of these days we’re going to have another one. Our salaries may not be so high nor our teaching loads as light as those cow colleges’, but we’ve got class.

A: Even if I don’t take the job, I’ll put a tiger in my tank. But just how big is the teaching load?

I1: Eleven hours.

A: Eleven hours a month isn’t too bad—after all, I run out of material on near-decomposability after 22 hours. But throw in a few trips to Washington, a week or two at the Bureau for decompression, Christmas in Miami, and the term is over.

I1: The Princeton faculty doesn’t go to Miami. I’m afraid it’s eleven hours a week?

A: You are kidding. How can anybody teach eleven hours a week and still keep up his ONR [Office of Naval Research] project, his NSF [National Science Foundation] grant, and his consulting for oil companies?

I2: The whole Minnesota department doesn’t teach 11 hours a week. Don’t be hasty, sir. We’ll buy you a Community Antenna Television set-up.

I1: Don’t listen to him. You don’t have to lecture for 11 hours a week. You can work off some of it by discussion with graduate students.

A: I don’t see why Princeton graduate students should be treated better than MIT students. What’s the pay?

I1: Eighty-five hundred.

A: Eighty-five hundred! Is that in 1954 dollars or something?

I2: In Berkeley a teaching assistant gets 8500 just for picketing.

A: You’re having [?] me on.

I1: I can see you’re not the Princeton type. Hardly anyone is.

A: How clever can you get? Well, gentlemen, thank you for dropping in. I’ll let you know in due course. Don’t call me, I’ll call you.

I2: By the way, could you tell me what you’re writing your thesis on and how far you’ve got?

A: None of your goddamn business. But if you must know, Kuh once said that one regression is worth a thousand words. I figure 35,000 words makes a pretty fair thesis, so I’m doing 35 regressions.

I1: On what?

A: On a computing machine, you dope. Now I’m afraid I have another appointment. I suppose some of the other students have agreed to see you. Miss Tapley will show you the way.

D.T.: Your next appointment is ready. The gentlemen from Harvard and Yale are waiting, the Wharton School has sent Albert Ando and two other people he claim are named Flend [Irwin Friend] and Klavis [Irving Kravis], there is a man from Northwestern who drove up in a Brink’s armored car he says is full of bills in small denominations, and the New York Knickerbockers claim they’ve picked the whole class in the draft.

 

Source: M.I.T. Libraries, Institute Archives and Special Collections. MIT Department of Economics Records. Box 2, Folder “GEA 1961-67”.

Image Source:From the Flying Car to the Giant R2-D2: The Greates MIT Hacks of All-Time“, by Robert McMillan. Wired, March 20, 2013.

“Boston’s Harvard Bridge is 364.4 Smoots long. And the fact that anybody would remember this in 2013 was probably the furthest thing from MIT freshman Oliver Smoot’s mind on the October 1958 night that he lay himself down, time and again, along the bridge, allowing his fraternity brothers to measure its length (each Smoot is about 5 feet, 7 inches). It was a fraternity prank, but the next year the bridge’s Smoot markers were repainted. Thus, an MIT landmark — and a unique unit of measurement — was born.

Smoot himself went on to become a board member of the American National Standards Institute — a standards man through and through.”

Categories
Funny Business M.I.T.

M.I.T. Economics Faculty Skit à la Rowan and Martin’s “Laugh-In”, December 1968

 

This post continues our series “Funny Business” that features successful and less-than-successful attempts at humor by economists. Reading one of these historical skits demands the reader to concede that the defense, “It seemed funny at the time,” might actually be valid for fifty year old jokes.  At the December 1968 Graduate Economics Association party the M.I.T. economics faculty offered its version of the wildly popular, frenetic comedy series “Rowan and Martin’s Laugh-In” (like “Sit-in”, get it? As I just said, “it seemed funny at the time”). 

For young and non-U.S. historians of economics, remote learning of the original Laugh-In content is easy:

Rowan & Martin’s Laugh-In information at IMDb.
Rowan & Martin’s Laugh-In highlights on YouTube.

The tag-line “Sock it to me” was a creation of the 1960s and made a meme by Rowan and Martin’s Laugh-In. Paul Samuelson closing the skit with that line is almost up there with 1968 Presidential candidate Richard Nixon’s saying it in his cameo appearance on Laugh-In.

The skit transcript below includes some square-bracketed comments to help the reader. Of course, nothing says “joke” more than a good footnote.

______________________

Reminder/Invitation

December 11, 1968

Graduate Students, Faculty Members
and Secretaries

DON’T FORGET!!

            A week from today is the GEA Christmas Party—Tuesday, December 17th. The festivities will begin at 8:00 pm in the Campus Room of Ashdown House. Admission is only $1.00 and the entertainment is free.

______________________

GEA CHRISTMAS SKIT 1968
[Faculty]

 

Music

[Franklin M.] Fisher: It’s the Faculty Laugh-In.

Music

(Enter [E. Cary] Brown, [Paul A.] Samuelson and [Robert L.] Bishop,
Brown and Samuelson sit.)

Samuelson: For the first question on your advanced theory oral:
Who was the greatest economist of all time?
Bishop (After much thought) Pigou…

Music

[Morris] Adelman: It is written: when offer curve bend backwards, then is time to send [Walt] Rostow to Texas.
[For background to Rostow Affair, see Appendix below]

Music—through

[Matthew D.] Edel (carries sign) “Economics is a dismal science”

([Peter] Temin and [Duncan] Foley enter as Rowan and Martin)

Foley: It certainly was a swell idea to put on a faculty laugh-in.
Temin: It’s so much easier than thinking up a connected skit.
Foley: Well, what cute laugh-in type feature do we have coming up next?
Temin: I see by my script here that we’re going to have a “Laugh-in looks at…” next.
Foley: Yes, it says: Faculty laugh-in looks at the new [Nixon] administration.

Music

[Jerome] Rothenberg: Washington: James Reston has expressed outrage at news reports that the University of Maryland has no plans to hire Spiro T. Agnew.
[Motivation for James Reston mention here see, Appendix “Rostow Affair” below]
Temin: Meanwhile at the Council of Economic Advisers, Republicans begin to grapple with the unaccustomed complexities of the Federal budget.

(enter Bishop and Foley)

Bishop: They always said Art Okun could do it with a pencil on the back of an envelope.
[See Appendix below]
Foley: I still think we’d better wait for the computer printout.
Bishop: No, look, its easy. Let’s see, how does it go? Is it Y = C + the deficit, or does the deficit = Y + C?

Music

Temin: At the same time we hear the swan song of liberals seeking sanctuary on college campuses.
Fisher: Song “Hey Dick [Nixon]”
[presumably to the tune of “Hey Jude”, lyrics to parody not in the file]
Rothenberg: Washington: the M.I.T. economics department has again startled Washington circles by announcing that it will not hire Henry Kissinger in 1972.
[cf. Appendix below on “Rostow Affair”]
Foley: Why don’t we just use their budget?
Bishop: And give up on the job? It can’t be that hard.
Foley: We don’t even have the computer printout yet.
Bishop: Doesn’t investment come in here someplace?

Music

Rothenberg: Washington: It has just been learned that the M.I.T. economics department, responding to the furor over the Rostow affair has abolished its economic history requirement.
[see Appendix below]

Music

(Man seated, knock on door: goes to answer, returns)

Adelman: Dear, Mr. Brower is here to fix the point (calling).
[Punny reference to Brower’s fixed-point theorem  that is a building block for the proof of the existence of a general equilibrium.]

Music—through

Edel (carries sign) “Pigou Power”

(Enter Bishop, Brown, Samuelson)

Brown: Describe an Edgeworth-Bowley Box.
Bishop: (gesturing) It’s about so wide…

Music

(Enter Foley and Temin)

Foley: What movie did you see last night?
Temin: “Thoroughly Modern Miltie”
[clearly “Milton Friedman”, the film’s title was “Thoroughly Modern Miltie”]

Music—through

Fisher (carries sign) “Nest principal minors”
[Linear algebra joke, written like a creepy, even pedophilic, command here, “nested principal minors” or “nest of principal minors” would be proper.]
Rothenberg: The negative definite is equivalent to the lie direct.
[Shakespeare As You Like It, V:iv in Appendix below]

Music

Foley: The computer printout is here!

(enter tons of printout)

Bishop: I think I’ve got it!
Foley: What?
Bishop: One of Okun’s envelopes. How old do you think this is anyway?

Music

Samuelson:

A Poem
by Paul A. Samuelson

Some people cover lots more ground
But no one handles the New York Times like Carey Brown.

[Likely another reference to the Rostow Affair, see Appendix Below]

Music

(Adelman seated, door knock)

Adelman: Dear, Mr. [Evsey] Domar is here to compare the systems.
[One of Evsey Domar signature courses was “Comparative Economic Systems”]

Music

Foley: What movie did you see last night?
Temin: Ride the high Pontry
[“Ride the High Country”, 1962 Western film by Sam Peckinpah]
Foley: What Pontry again?
[A punny reference to Pontryagin’s maximum principle in optimal control theory.]

Music

(Enter Bishop, Samuelson, Brown)

Brown: What was Marshall’s greatest contribution?
Bishop: In 1903, Marshall gave £1500 to King’s College.

Music

(Enter Fisher and Temin with box)

“2 squares least stage”
(sign)
[“2-stage least squares” is the name of statistical procedure, here Fisher and Temin are the two “squares“.]

Music

Adelman: Mark Hopkins said the ideal education is a professor and a student sitting on a log, with the professor talking to the student. I sometimes think I would get the same results sitting on the student and talking to the log.

Music

Bishop: Sock it to me

Music

(Enter Temin and Foley)

Temin: Here we are out here again imitating Rowan and Martin.
Foley: Shouldn’t you be standing on the other side? What now?
Temin: Now we’re giving the “Flying Fickle Finger of Fat Award” just like on TV.
Foley: And who gets the “Flying Fickle Finger of Fat Award”?
Temin: Fate. The Flying Fickle Finger of Fate Award goes to…

(Music cue—fanfare)

Temin: Kenneth Boulding for receiving a vote of confidence from…himself.
[Boulding gave his Presidential address to the American Economic Association a few weeks later on “Economics as a Moral Science”. For likely background to the joke see the Appendix below.]

Music

Fisher: A Bordered hessian is a German mercenary surrounded by continentals.

Music

Samuelson:

(carries sign) “I am an external economist.”

Music

Foley: What movie did you see last night?
Temin: “Closely watched brains”
[“Closely watched trains”, 1966 Czech film directed by Jiří Menzel]

Music

Foley: (Poring over computer printout). I think the whole idea of the budget is a stupid, dumb, stupid idea. Why do we even need a budget?
Bishop: Look, we’ve got to have something to send down to the Congress tomorrow.
Foley: I’m going to hold my breath until the stupid deficit comes out right.
Bishop: Just try to remember whether capital gains are part of income or not.

Music cue

(Enter Fisher, Temin, Edel)
“3 squares least stage”
(sign)
[“3-stage least squares” is a statistical procedure, and Fisher, Temin and Edel are the three “squares“.]

Music

Brown: The students are revolting.
Bishop: Yes, I’ve though so for a long time.

Enter Everybody

Rothenberg: SDS Sam
[SDS=Students for a Democratic Society…
(wild guess) impression of Bogart saying “Play it Again Sam”?]
Foley: Well, here we are out here again, and it’s time to say…
Temin: Long joke.
Foley: Say goodnite, Peter.
Temin: Goodnite, Peter.
Samuelson: Sock it to me.

Source: M.I.T. Archives.  Folder “GEA 1967-68”.

_________________________

Appendix

 

Rostow Affair

Source: Howard Wesley Johnson, Holding the Center: Memoirs of a Life in Higher Education. From Chapter 8, pp. 189-90.

*   *  *  *  *  *  *  *  *  *  *  *

 

Art Okun’s Reputation as an economic forecaster “on the back of an envelope”

Source: Joseph A. Pechman contribution for In Memoriam: Arthur M. Okun. November 28, 128–March 23, 1980 (Washington, D.C.: Brookings Institution, 1980), p. 14.

*   *  *  *  *  *  *  *  *  *  *  *

 

From Shakespeare’s As You Like It
Act V, Scene 4.

JAQUES

Can you nominate in order now the degrees of the lie?

TOUCHSTONE

O sir, we quarrel in print, by the book; as you have
books for good manners: I will name you the degrees.
The first, the Retort Courteous; the second, the
Quip Modest; the third, the Reply Churlish; the
fourth, the Reproof Valiant; the fifth, the
Countercheque Quarrelsome; the sixth, the Lie with
Circumstance; the seventh, the Lie Direct. All
these you may avoid but the Lie Direct; and you may
avoid that too, with an If. I knew when seven
justices could not take up a quarrel, but when the
parties were met themselves, one of them thought but
of an If, as, ‘If you said so, then I said so;’ and
they shook hands and swore brothers. Your If is the
only peacemaker; much virtue in If.

Source: From the Shakespeare homepage at M.I.T.

*   *  *  *  *  *  *  *  *  *  *  *

 

Kenneth Boulding’s Vote for AEA to Meet in Chicago in 1968

 

Source:  Robert Scott, Kenneth Boulding: A Voice Crying in the Wilderness (Palgrave Macmillan, 2014).

 

 

Categories
Exam Questions Johns Hopkins Suggested Reading Syllabus

Johns Hopkins. Reading List and Exam for Aggregate Income Theory. Machlup, 1951

 

Materials (reading list and exams) for Fritz Machlup’s course on income distribution, 18-603, have been transcribed and posted earlier. Economics in the Rear-view Mirror also has a transcription of the final exam for his 1956 course on methodology.

________________________

 

Course Announcement

Theory of Aggregate Income 604. Professor Machlup.
Two hours weekly, second term.

A study of the theory of income formation, linking an analysis of the supply and circulation of money with a dynamic process analysis of autonomous and induced disbursements for consumption and investment; an attempt to explain the level and fluctuations of national income.

Source: Johns Hopkins University. School of Higher Studies of the Faculty of Philosophy, Announcements of Courses 1950-51 (The Johns Hopkins Circular, April 1950), p. 99.

________________________

 

JOHNS HOPKINS UNIVERSITY
THE THEORY OF AGGREGATE INCOME
18-604

Prof. Fritz Machlup

READING LIST
Spring Term 1951

Books:

Required:

J. M. Keynes, The General Theory of Employment, Interest and Money. (London: Macmillan, New York: Harcourt Brace, 1936) pp. 1-384.

Recommended:

Richard Ruggles, An Introduction to National Income and Income Analysis. (New York: McGraw-Hill, 1949)

Thomas C. Schelling, National Income Behavior. (New York: McGraw-Hill, 1951)

A.E.A., Readings in Business Cycle Theory. (Philadelphia: Blakiston, 1944)

Seymour E. Harris (ed.), The New Economics. New York: Knopf, 1947)

 

I. Static and Dynamic Analysis

Paul A. Samuelson, “Dynamic Process Analysis,” A Survey of Contemporary Economics, ed. Howard S. Ellis. (Philadelphia: Blakiston, 1948) pp. 352-387.

 

II. Savings, Investment, and National Income

Bertil Ohlin, “Some Notes on the Stockholm Theory of Saving and Investment,” Readings in Business Cycle Theory, pp. 87-131.

Friedrich A. Lutz, “The Outcome of the Saving-Investment Discussion,” Readings in Business Cycle Theory, pp. 131-157.

Abba P. Lerner, “Saving and Investment: Definitions, Assumptions, Objectives,” Readings in Business Cycle Theory, pp. 158-168.

Oscar Lange, “The Rate of Interest and the Optimum Propensity to Consume,” Readings in Business Cycle Theory, pp. 169-192.

Fritz Machlup, “Forced or Induced Saving,” The Review of Economics and Statistics, Vol. 25 (1943), pp. 26-39.

Dennis H. Robertson, “A Survey of Modern Monetary Controversy,” Readings in Business Cycle Theory, pp. 311-329.

 

III. The Multiplier

Fritz Machlup, International Trade and the National Income Multiplier (Philadelphia: Blakiston, 1943) Chapters 1-7, 10.

Gottfried Haberler, “Mr. Keynes’ Theory of the ‘Multiplier’: A Methodological Criticism,” Readings in Business Cycle Theory, pp. 193-202.

Fritz Machlup, “Period Analysis and Multiplier Theory,” Readings in Business Cycle Theory, pp. 203-234.

Robert Eisner, “The Invariant Multiplier,” Review of Economic Studies, Vol. 17 (1949-50), pp. 198-202.

 

IV. Velocity and Time Lags

James W. Angell, “The Components of Circular Velocity of Money,” Quarterly Journal of Economics, Vol. 51 (1937), pp. 224-272.

Lloyd Metzler, “Three Lags in the Circular Flow of Income,” Income, Employment, and Public Policy. (New York: W.W. Norton and Company, 1948), pp. 11-32.

Alvin H. Hansen, “The Robersonian and Swedish Systems of Period Analysis,” Review of Economics and Statistics, Vol. 32 (1959), pp. 24-29.

Harold M. Somers, “A Theory of Income Determination,” Journal of Political Economy, Vol. 58 (1950), pp. 523-541.

 

V. Wage Rate Reductions and Employment

A. C. Pigou, “Real and Money Wage Rates in Relation to Unemployment,” Economic Journal, Vol. 47 (1937), pp. 405-422.

N. Kaldor, “Professor Pigou on Money Wages in Relation to Unemployment,” Economic Journal, Vol. 47 (1937), pp. 745-762.

A. C. Pigou, “Money Wages in Relation to Unemployment,” Economic Journal, Vol. 48, (1938), pp. 134-137.

James Tobin, “Money Wage Rates and Employment”, in The New Economics, pp. 572-587.

 

VI. Tax-Financed Government Expenditures

Trygve Haavelmo, “Multiplier Effects of a Balanced Budget,” Econometrica, Vol. 13 (1945), pp. 311-318.

Gottfried Haberler, “Multiplier Effects of a Balanced Budget: Some Monetary Implications of Mr. Haavelmo’s Paper,” Econometrica, Vol. 14 (1946), pp. 148-149.

R. M. Goodwin, “Multiplier Effects of a Balanced Budget: The Implications of a Lag for M. Haavelmo’s Analysis,” Econometrica, Vol. 14 (1946), pp. 150-151.

Everett E. Hagen, “Multiplier Effects of a Balanced Budget: Further Analysis,” Econometrica, Vol. 14 (1946), pp. 152-55.

T. Haavelmo, “Multiplier Effects of a Balanced Budget: Reply,” Econometrica, Vol. 14 (1946), pp. 156-58.

 

VII. The Accelerator

John M. Clark, “Business Acceleration and the Law of Demand: A Technical Factor,” Readings in Business Cycle Theory, pp. 235-260.

Paul A. Samuelson, “Interactions Between the Multiplier Analysis and the Principle of Acceleration,” Readings in Business Cycle Theory, pp. 261-269.

F. A. Hayek, Profits, Interest, and Investment. (London: Routledge and Sons, 91939) Chapter I, pp. 3-72.

F. A. Hayek, “The Ricardo Effect,” Economica (1942), pp. 126-152.

 

Source: Duke University. David M. Rubenstein Rare Book and Manuscript Library. Economists’ Papers Archive. Evsey D. Domar Papers, Box 15, Folder “Macroeconomics: Old Reading Lists”.

 

________________________

 

THE JOHNS HOPKINS UNIVERSITY
Theory of Aggregate Income
(18-604)
Final Exam, May 29, 1951

Professor Fritz Machlup

Answer three questions, one of each group, as concisely as possible without omitting significant steps in your reasoning. Write in English rather than in algebra or geometry.

I.

  1. Explain the possibilities of a general cut in money wage rates bringing about an increase in aggregate employment.
  2. Explain the increase in employment that can be brought about by an increase in tax-financed government expenditures, emphasizing alternatively the significance in the causal sequence of changes in (a) the quantity of money or velocity of circulation, (b) liquidity preference, (c) the propensity to consume, (d) the difference between investment and saving.

 

II.

  1. Explain the meaning of the distinctions between the “consumption lag”, the “output lag”, and the “earnings lag”, and their importance, or lack of importance, in the determination of income.
  2. Explain the meaning of the distinctions between “intended” and “unintended” saving, and “intended” and “unintended” investment, and their importance, or lack of importance, in the determination of income.

 

III.

  1. Explain the interactions between multiplier and accelerator.
  2. Explain the meaning of the “Ricardo Effect” and its importance, or lack of importance, in the determination of the accelerator and of the turning points of the business cycle.

 

Source: The Johns Hopkins University. The Eisenhower Library, Ferdinand Hamburger, Jr. Archives. Department of Political Economy, Series 6, Box 3/1, Folder “Graduate Exams, 1933-1965”.

Image Source: Fritz Machlup is seen presenting in a seminar (note: Evsey Domar is leaning forward on the right side of the table, third from the left). From the Johns Hopkins Yearbook Hullabaloo 1956, p. 15.

Categories
Funny Business M.I.T.

M.I.T. Dystopian Faculty Skit by Solow,1969

 

 

The current events of the late ‘sixties are the clear inspiration for this somewhat dark, dystopian skit for the M.I.T. economics departmental Christmas party of December 1969. According to the cover page, it was written by Robert Solow with input from Frank Fisher.

The skit was transcribed from the typed text [that includes a short handwritten addition] from Robert Solow’s papers in the Economists’ Papers Archive at Duke University. A grateful tip of the hat to Roger Backhouse for this artifact that should keep a cultural historian of economics busy for a few hours and be worth a few minutes of procrastination for working economists.

 

Pro-tip: you can summon all of the Economics in the Rear-view Mirror posts with economic humor content using the keyword “Funny Business”:

https://www.irwincollier.com/category/funny-business/

_______________________

Back-story for selected references in the text

SPECTRE. In Ian Fleming’s world of James Bond the acronym for the organization of international evil [Special Executive for Counter-intelligence, Terrorism, Revenge and Extortion].

Chairman Edel. Assistant Professor Matthew D. Edel (Yale, Ph.D.) taught the course Economic Growth and Development. Presumably pronounced to rhyme with “Fidel”. Edel was a regional expert for Latin America, spoke at a colloquium February 4, 1970 on “The Strategy of Cuban Economic Development

14.463 Monetary Economics in term I, 1969-70 was taught by four instructors.

According to the staffing report for that term in the departmental records at the MIT archive.

Karen H. Johnson, M.I.T. Ph.D. (1973),
Robert K. Merton, M.I.T. Ph.D. (1970), advisor Paul Samuelson
David T. Scheffman, M.I.T. Ph.D. (1971), advisor Paul Samuelson
Jeremy J. Siegel, M.I.T. Ph.D. (1971)

There is no record that Bonnie Parker and Clyde Barrow were ever graduate students of economics in M.I.T.

Bread and Roses. Reference to the Women’s Liberation Organization in Boston, 1969-1971. The name chosen in memory of the Great Lawrence Strike of 1912.

Ted Behr. An M.I.T. Ph.D. (1969) who by 2009 had already gone through seven career changes and twelve jobs. Must have been quite a character judging from this interview.

I think we may assume that no Bulgarians were injured in the writing or performance of this skit.

_______________________

Some Obvious Context

Fall 1964. Berkeley Free Speech Movement

Wikipedia Entry on the Protest Year 1968

April 1968. Columbia Student Strike ; Harvard Student Strike

February 1969. Black student strike at the University of Wisconsin

_______________________

RIP VAN SAMUELSON RETURNS TO MIT AFTER THE REVOLUTION
FACULTY SKIT
Christmas 1969

CAST

P. Diamond
R. Eckaus
R. Engle
F. Fisher
C. Kindleberger
M. Piore

SCRIPTWRITER-IN-CHIEF — R. Solow

HELPED BY – F. Fisher

Is it really true that Samuelson has been asleep all these years? Then how come the 13th and 14th editions of the textbook came out on time?

Well, I don’t know. Samuelson isn’t talking.

Careful, there. If it’s not talking it’s not Samuelson.

It’s got to be. His broker recognizes his fingerprints from soiled sell orders. Actually, there are two schools of thought about how the textbook came out while Samuelson was sleeping. Modigliani claims that the 13th and 14th editions were simply forecasted by the FRB-MIT model, using a long lag. But some people believe that the 13th and 14th editions are just the 2nd and 3rd editions reprinted. Can’t verify that, though. Nobody’s been able to find a copy of the early editions.

Not that it matters. Must be a shock for Paul to realize that nobody uses the text any more, except of course for the Bulgarian translation. They’re the only people reactionary enough to go for that stuff any more.

You mean even Hanoi University has dropped it?

Oh sure, they adopted Best Known Thoughts of Chairman Edel, last year. You know, the one that begins “Equilibrium grows out of a barrel…”

Out of the barrel of a gun?

No, no, a barrel of rum. Chairman Edel never got over that trip to Cuba.

Did you fellows hear that Samuelson is back? When did he disappear anyway?

Oh, a long time ago. Even before Chomsky became President. It’s hard to know the exact date. Things were pretty clear up until April 1972, when we were supposed to have 31 days of moratorium, but the month only had 30 days, so we cancelled the first day of May, only you couldn’t cancel May Day — Christmas you could cancel, but not May Day. So we cancelled the second day of May. But then we were three days short to fit in the 32 days of moratorium for that month, so we had to run into June. From then on it was chaos.

Things are still a little funny. I can’t get used to having summer vacation in the middle of winter, and Fisher pretending to go off skiing when it’s 90 degrees in the shade, when we all know he’s leading rent strikes anyway.

Don’t complain. It might have been worse. Solow claimed to have a proof that the term would never end once we got up to 32 moratorium days a month. But one of the younger mathematical economists made a brilliant application of the theory of Riemann surfaces and showed that you could pack any finite number of moratorium days into one month if you did it right.

It was the last article anyone published in this department. Can you remember when we used to write articles and hope for tenure? That was before tenure was abolished. God, life was easy then. Nowadays it’s all action, action, action. And if you’re lucky, if you happen to win a rent strike, or destroy some draft records, or win an amateur topless contest, then maybe the central committee of SPECTRE will keep you on for a year. But suppose you lose the strike, or you let a white man go to work on a construction site, boy that SPECTRE can be tough. You remember when they threw Domar into the arena with Kampf and gave Kampf the bullhorn?

I looked away. Bloodthirsty crew — they awarded Kampf both ears and the tail that day. We had to take up a collection to send Ricky and Alice [note: Evsey Domar’s daughters] to Bread and Roses Karate School. And today they’re members of SPECTRE, the Student Power Electoral Committee for Teachers of Relevant Economics. It was better in the old days when appointments went on good looks and amiability. Even publishing was better than action all the time. That last piece of work I did, keeping the recruiter for Mars Bars from getting onto the campus, it went well but it was exhausting.

Why are we against Mars Bars?

Space, military, it’s all the same.

Anyhow, now that he’s back, what’s Paul going to do around the department? He’s getting a little old for real action, and he might find it hard to pass the monthly Relevance Check.

It’s going to be a problem. He was falling behind the times when he went to sleep. Of course he looks better now, with 10-15 years growth of beard, but he doesn’t dig the revolution. El Lider Maximo of the Graduate Student Commune asked him what he could contribute, and Samuelson said he’d like to teach the History of Economic Thought.

The History of WHAT???

That’s exactly what the Commune Lider said.

Poor old Samuelson doesn’t know that Thought isn’t Relevant. In fact he didn’t even know that Economics isn’t Relevant. When El Lider explained that it was all action now, old Samuelson said he thought there should be both Thought and Action just so their marginal net productivities were equal.

Gad, I haven’t heard anything like that since the day they fired Diamond for saying “Pareto-optimal” once too often.

Whatever happened to Diamond?

What else, he’s at B.I.T., the Bulgarian Institute of Technology. Boy, if the old stuff ever comes back in style, those Bulgarians will have it made. But go on, what happened when Samuelson pulled that bourgeois bit about marginal whatnots?

Well, Solow was standing there and he muttered something to Samuelson—it sounded like “Check the second-order conditions, Paul old boy”—and then went back to trying to look hip.

That’s living dangerously.  Solow just barely passed last month’s Relevance Check, and he hasn’t been on a successful action in a long time. I don’t think that went over so good when he claimed that skiing Black Mountain was a real action. He better watch out — if B.I.T. won’t take an old man like that, SPECTRE may throw him to Kampf.

Right on. Nothing gets past El Lider. When Solow whispered that to Samuelson about second-order conditions, El Lider asked him right away — Did you say something? Solow replied Negative. Definite. That’s really living dangerously — I think it’s code of some kind.

It certainly doesn’t sound Relevant. I haven’t read anything like that in Ted Behr’s Newsweek column, at least not lately.

What’s going on this week in the department?

In the Theory course we’re holding an obstructive picket line at the drug counter of the Tech Store. Somebody discovered they were selling only white pills.

If I know what the pills are for, I hope the picket line isn’t too obstructive.

Of course not; I told you it was the Theory course. Then in the Economics of Education course we’re going to burn down a school. In the Money course, Johnson, Merton, Siegel, Bonnie, and Clyde are going to rob a bank and distribute the proceeds to the C.L.F.

Is that the California Liberation front?

Oh no, Berkeley has been a free-fire zone for months; nobody is left. It’s the Center for Love and Finance, our answer to the profit motive. Has anyone told you what the Econometrics Commune is doing?

No. Last week somebody had an idea for an empirical paper, but the results only came out at the 10% Relevance Level and half the commune was purged for Type One Error.

Served them right. Any Type II Error executions?

You know we have to have public trials for Type II error.

That’s right—Power to the People…. Well, it’s nice to see that the action curriculum is moving along. Sure beats the Old Days before chairman Edel — remember when they taught about Indifference curves? INDIFFERENCE curves, mind you, with innocent people being napalmed in Laos, Birmingham, Princeton, they taught about indifference curves.

Hard to believe. Of course now, ever since we adopted Bohmer’s best-selling text Economics for Good Guys we handle all that stuff by the tangency of the Relevance Map and the Isoconcern lines. Makes all the difference in the world, takes the subject out of the mind and puts it back in the gut, where it obviously belongs.

The Admissions Commune has been meeting all day.

How does the entering Movement look?

Terrific. There’s one girl who was heavyweight sugar-cane-cutting champion of the Big Ten, and another who had already led three successful rent strikes as a junior — two of them publishable, according to her advisor. Then there are a couple of Black Belts from Bread and Roses — they come on Karate Scholarships of course.

Any amateur topless contest winners?

We’re trying for a few, but most of them will go to Harvard—ever since they hired Brigitte Bardot for the economics faculty—

She was past her peak.

Peaks. And aren’t they all? Anyhow, all the amateur topless winners go to Harvard. But we’ve got some applicants who’ve starred in home movies. Not to mention a few school-burners and a couple of guys who have specialized in destroying computers.

How are their vibrations?

Good.

Fine. If there’s anything I can’t stand it’s bad vibrations. How about GRE scores.

The Graduate Relevance Exam grades just came — most of the people we’re accepting are in the 800’s on Obstructive and at least 750 in Vituperative. Looks like a good class — I mean Movement.

Has anyone heard what the Placement and Appointments Committees have decided?

They decided to eliminate the middleman and merge. That way everybody stays forever — once a Commune always a Commune. It gives new meaning to that old phrase about departmental inbreeding.

We still have this problem about what to do with Samuelson. Here he is after all those years asleep and hardly knowing anything about action and relevance and all the new things. The Bulgarians won’t take him — B.I.T. doesn’t mind using the old textbook, but they’re overloaded with these old-timers. If we can’t find something for him to do we may have to throw him to….

Terrible news. The students are revolting again. There’s a new movement sweeping all the Communes. They want one day of classes this month, two days of classes next month, three days the month after…there’s no telling where it will end, except that nobody can count over 30 any more.

Gad, we may have to go back to teaching again. Well, at least that gives something for Samuelson to do.

Oh didn’t they tell you. When Samuelson saw what the new system was like, he went back to sleep. Better get the Bulgarians on the phone.

 

Source: Duke University. David M. Rubenstein Rare Book and Manuscript Library. Economists’ Papers Archive. Papers of Robert M. Solow, Box 83.

Image Source: Robert Solow in his office, MIT Museum Website.

Categories
Exam Questions Johns Hopkins M.I.T. Suggested Reading Syllabus

M.I.T. Readings and exam questions for fiscal and monetary policy. Domar, 1957

Evsey Domar’s first semester at M.I.T. was as a visiting professor according to the teaching records of the economics department. He taught one seminar on Russian Economics (14.292) and a graduate course with the nominal title “Fiscal Policy”. That course had been taught previously by E. Cary Brown (Spring 1954, 1955) and R. A. Musgrave, visiting Professor (Spring 1956).

Inspection of the ten-page course bibliography and the final examination questions along with two note-cards filed with these course materials, it appears that well over half the course was in all likelihood dedicated to fiscal policy topics with monetary policy for stabilization topics accouting for perhaps one-third of the course. Just as the length of the course bibliography (typical for Domar) is daunting, his use of asterisks to designate recommended reading was exceedingly liberal. An examination of the final examination questions leads me to conclude that it should be rather easy to reduce the course reading list (for examination purposes!) to less than two pages.

___________________________

Course Enrollment
(Second term, 1956-57

Instructor

Domar, E. D.

Rank

Prof. (Visit.)

Subj. No.

14.472

Subj. Title

Fiscal Policy

No. Class Hours/Week

3

No. Students

22

Source: M.I.T. Archives, Department of Economics Records 1947-, Box 3, Folder “Teaching Responsibility”.

___________________________

Typed notecards for an introduction to or a review of course.

The traditional arguments regarding the purposes of Monetary Policy:

  1. Stabilization of general prices or of factor earnings—the Wicksell-Davidson controversy. The instrument was the relation between the natural and the market rates.
  2. Stabilization of prices or of employment. Recent literature is full of this.
  3. Stabilization of the general prices or of prices of Federal securities. See Douglas’s and other reports on this recent controversy.
  4. Stabilization of employment or the achievement of growth. Any conflict?
  5. Discretionary methods or automatic provisions? See Simons’ article in Readings in Monetary Theory.
  6. To provide credit and currency, sound and in sufficient quantity.
  7. To protect the international position of the country.
  8. To have special effects, such as:

a. by region
b. by industry
c. by commodity consumed (such as tobacco) or housing
d. on population (by giving exemptions or subsidies for dependents)

  1. Provide revenue [handwritten addition]
  2. Distribution of income [handwritten addition]

*  *  *  *  *  *  *  *  *  *  *  *  *

The following limitations, some real, other imaginary, explain why Fiscal Policy is not as simple as Lerner makes it:

Income distribution
Size of the deficit
Size of the budget
Balance of payments
Special regional and industrial effects
Effects on incentives to work (in inflation)
Automaticity of the system (built-in-flexibility)
Monetary effects (on reserves, deposits)
Long-run effects (on growth and development)

Their presence complicates things and explains all the ingenious articles and tax devices frequently suggested. If not for them, fiscal policy would be very simple indeed: cut taxes or increase taxes, and the same with expenditures.

___________________________

READING LIST
14.472 Fiscal Policy
Spring Term 1956-57

Professor E. D. Domar

PART I—MONETARY POLICY

The purpose of this list is to suggest to the student the sources in which the more important topics in Monetary Policy are discussed from many points of view. His objective should be the understanding of these topics and not the memorization of who said what.

Most of the sources listed here, and particularly the Congressional materials, discuss a number of questions not only in Monetary but in Fiscal Policy as well. Hence it is difficult to classify them.

Items marked with an * are strongly recommended. (I don’t like to use the expression “required” in a graduate reading list.)

  1. Factual Materials on Monetary Problems

Federal Reserve Bulletin.

Treasury Bulletin.

Annual Reports of the Secretary of the Treasury and of the Board of Governors of the Federal Reserve System.

Historical Statistics of the United States, 1789-1945, and the Continuation to 1952.

Congressional Hearings, Reports and other Materials listed below.

  1. Introduction

Hart, Albert Gailord, Money, Debt and Economic Activity, New York 1948.

Hicks*, J. R., “A Suggestion for Simplifying the Theory of Money,” Economica, 1935; reprinted in Readings in Monetary Theory.

Lerner*; Abba P., “Functional Finance and the Federal Debt,” Social Research, 1943, and Readings in Fiscal Policy, p. 468, also Chapter 24 in his Economics of Control, New York, 1944.

Poole, Kenyon E., ed. Fiscal Policies and the American Economy.

Sproul* Allan, “Changing Concepts of Central Banking,” Money, Trade and Economic Growth in Honor of John Henry Williams, New York, 1951.

  1. Monetary Theory and Growth

Gurley*, John G. and Shaw, E. S., “Financial Aspects of Economic Development,” American Economic Review, September 1955, pp. 515-538.

  1. Effectiveness of the Interest Rate

Ebersole*, J. F., “The Influence of Interest Rates,” Harvard Business Review, XVII, i, 1938, pp. 35-39.

Henderson*, R. D., “The Significance of the Rate of Interest,” Oxford Economic Papers, October 1938, I, pp. 1-13.

*Meade, J. E. and Andrews, P. W. S., “Summary of Replies to Questions on Effects of Interest Rates,” Oxford Economic Papers, October 1938, I, pp. 14-31.

Sayers, R. S. “Business Men and the Terms of Borrowing,” Oxford Economic Papers, Feb. 1940, III, pp. 23-31.

Andrews, P. W. S., “A Further Inquiry into the Effects of Rates of Interest,” Oxford Economic Papers, Feb. 1940, III, pp. 32-73.

White*, William H., “Interest Inelasticity of Investment Demand—The Case from Business Attitude Survey Re-Examined,” American Economic Review, September 1956, pp. 565-87.

Lutz, Friedrich A., “The Interest Rate and Investment in a Dynamic Economy,” American Economic Review, Dec. 1945.

  1. General Surveys of Monetary Policy

Federal Reserve Board*, Tenth Annual Report for 1923. See pp. 29-39 particularly.

Chandler*, Lester V., “Federal Reserve Policy and the Federal Debt,” American Economic Review, 1949, and Readings in Monetary Theory, p. 394.

Hardy, Charles O., “Fiscal Operations as Instruments of Economic Stabilization,” American Economic Review, Supplement, 1948, pp. 395-403 and Readings in Monetary Theory, p. 394.

Hart, Albert Gailord, “Monetary Policy for Income Stabilization,” Income Stabilization for a Developing Democracy, ed. by Max F. Millikan, New Haven, 1953.

Williams, John H., “The Implications of Fiscal Policy for Monetary Policy and the Banking System,” AER Proceedings, March 1942; Readings in Fiscal Policy, p. 185.

Smith*, Warren L., “On the Effectiveness of Monetary Policy,” American Economic Review, September 1956, pp. 588-606.

  1. Suggested Objectives and Policies

Hammarskjold, Dag, “The Swedish Discussion on the Aims of Monetary Policy,” reprinted in International Monetary Papers, No. 5, pp. 145-154.

Simons*, Henry C., “Rule versus Authorities in Monetary Policy,” JPE, 1936, and Readings in Monetary Theory, p. 337.

Simons, Henry, “On Debt Policy,” JPE, Dec. 1944, and Readings in Fiscal Policy.

Mints*, Lloyd, W., “Monetary Policy,” Review of Econ. and Stat., 1946 and Readings in Fiscal Policy, p. 344.

Bach*, G. L., “Monetary-Fiscal Policy Reconsidered,” JPE, Oct. 1949, and Readings in Fiscal Policy.

Friedman*, Milton, “A Monetary and Fiscal Framework for Economic Stability,” American Economic Review, 1949, and Readings in Monetary Theory, p. 369.

*United Nations. National and International Measures for Full Employment. Report by a group of experts appointed by the Secretary-General (Lake Success, New York, December 1949).

Viner*, Jacob, “Full Employment at Whatever Cost,” QJE, August 1950, pp. 385-407. Reproduced with omissions in Economic Policy, Readings in Political Economy, edited by William D. Grampp and Emanuel T. Weiler, Homewood, Ill., 1956, pp. 54-65.

Samuelson* Paul A., “Principles and Rules in Modern Fiscal Policy: A New-Classical Reformulation,” Money, Trade and Economic Growth in Honor of John Henry Williams, New York, 1951.

Seltzer* Lawrence H., “Is a Rise in Interest Rates Desirable or Inevitable,” American Economic Review, Dec. 1945; Readings in Fiscal Policy, p. 202.

Roosa, Robert V., “Interest Rates and the Central Bank,” Money, Trade and Economic Growth in Honor of John Henry Williams, 1951.

Roosa*, Robert V., “Integrating Debt Management and Open Market Operations,” American Economic Review, 1952, and Readings in Fiscal Policy, p. 265.

Hansen*, Alvin H., “Monetary Policy,” The Review of Economics and Statistics, May 1955, pp. 110-119.

  1. Commodity Money

Graham, Benjamin, World Commodities and World Currency, New York 1944.

Graham*, Frank D., “Full Employment without Public Debt, Without Taxation, Without Public Works, and without Inflation,” Planning and Paying for Full Employment, edited by Abba P. Lerner and Frank D. Graham, 1946.

  1. Congressional Materials

Joint Committee on the Economic Report. Money, Credit, and Fiscal Policies. Hearings before the Subcommittee on Monetary, Credit and Fiscal Policies of the Joint Committee on the Economic Report, 81st Congress, First Session, September 23, November 16,17,18,22,23 and December 1,2,3,5,7, 1949.

Joint Committee on the Economic Report. Monetary, Credit, and Fiscal Policies. A Collection of Statements Submitted to the Subcommittee on Monetary, Credit and Fiscal Policies by Government Officials, Bankers, Economists, and Others. 1949.

Joint Committee on the Economic Report (The Douglas Subcommittee). A Compendium of Materials on Monetary, Credit, and Fiscal Policies. A Collection of Statements Submitted to the Subcommittee on Monetary, Credit, and Fiscal Policies by Government Officials, Bankers, Economists, and Others. 81st Congress, 2ndSession, Senate Document No. 132, 1950.

Joint Committee on the Economic Report*. Monetary, Credit, and Fiscal Policies. Report of the Subcommittee on Monetary, Credit, and Fiscal Policies of the Joint Committee on the Economic Report. 81st Congress, 2ndSession, Senate Document No. 129, 1950.

Joint Committee on the Economic Report. Monetary Policy and the Management of the Public Debt Hearings before the Subcommittee on General Credit Control and Debt Management of the Joint Committee on the Economic Report, 81st Congress, 2nd Session, March 1952.

Joint Committee on the Economic Report. Monetary Policy and the Management of the Public Debt. Their Role in Achieving Price Stability and High-Level Employment. Replies to questions and other material for the use of the subcommittee on general credit control and debt management. 82nd Congress, 2nd Session, Senate Document No. 123, 1952.

Joint Committee on the Economic Report. Monetary Policy and the Management of the Public Debt Report of the Subcommittee on General Credit Control and Debt Management of the Joint Committee on the Economic Report, 82nd Congress, 2nd Session, 1952.

Joint Committee on the Economic Report. United States Monetary Policy: Recent Thinking and Experience Hearings before the Subcommittee on Economic Stabilization of the Joint Committee on the Economic Report. 83rd Congress, 2nd Session, December 6 and 7, 1954.

Joint Committee on the Economic Report. January 1956 Economic Report of the President. Hearings before the Joint Committee on the Economic Report. 84th Congress, 2nd Session, January 31, February 1,2,3,6,7,8,9,14,15,17 and 28, 1956.

Joint Committee on the Economic Report*. Conflicting Official Views on Monetary Policy; April 1956. Hearings before the Subcommittee on Economic Stabilization of the Joint Committee on Economic Report, 84thCongress, 2nd Session, June 12, 1956.

  1. Readings for Amusement

Outside Readings in Economics, second edition. Selected by Hess, Arleigh P. Jr., Gallman, Robert E., Rice, John P., and Stern, Carl, New York, 1956. The “Dialogue on Money,” by D. H. Robertson; “The Island of Stone Money,” by William H. Furness III; “The Paper Money of Kubla Khan,” by Marco Polo; and “The Edict of Diocletian,” by Humphrey Mitchell, pp. 314-335 are very amusing and instructive.

 

PART II—FISCAL POLICY

See the remarks in Part I.

  1. Factual Materials of General Character

Joint Committee on the Economic Report.* The Federal Revenue System: Facts and Problems, 1956

Treasury Bulletin

Annual Reports of the Secretary of the Treasury and of the Commissioner of Internal Revenue

Statistical Abstract of the United States

Historical Statistics of the United States, 1789-1945 (published by the U. S. Bureau of the Census)

U. S. Treasury Department, Internal Revenue Service, Statistics of Income (an annual publication in two volumes)

U. S. Bureau of the Census, Summary of Governmental Finances (annual series)

The Budget of the U. S. Government

Commerce Clearing House, Inc., Tax Systems

West Publishing Co., Federal Tax Regulations, 1956

Congressional Hearings and Reports, listed in Part I and below

Textbooks on Public Finance and Fiscal Policy

  1. Historical Studies

Ratner, S., American Taxation: Its History as a Social Force in a Democracy, New York, 1942

Fabricant*, S., The Trend of Government Activity in the United States since 1900, New York, 1952, Chapters 1, 6, 7

Studenski, P. and H. E. Kroos, Financial History of the United States, New York, 1952

Musgrave*, R. A. and J. M. Culbertson, “The Growth of Public Expenditures in the United States,” National Tax Journal, June, 1953, pp. 97-115

Paul, R. E., Taxation in the United States, Boston, 1954

  1. Fundamental Assumptions

Hansen*, A. H., “The Stagnation Thesis,” Fiscal Policy and the Business Cycle, New York, 1941, pp. 38-46, and Readings in Fiscal Policy

Schumpeter*, J. A., “Economic Possibilities in the United States,” Capitalism, Socialism, and Democracy, 1947, and Readings in Fiscal Policy

Domar*, E. D., “The Problem of Capital Accumulation,” The American Economic Review, December, 1948

Fellner*, W., “Relative Emphasis in Tax Policy on Encouragement of Consumption or Investment,” Federal Tax Policy for Economic Growth and Stability, Joint Committee on the Economic Report, Washington, D.C., November 9, 1955, p. 210

Hansen*, A. H., “Economic Stability and Growth,” Federal Tax Policy for Economic Growth and Stability, Joint Committee on the Economic Report, Washington, D. C., November 9, 1955, p. 14

Smithies*, A., “Economic Growth as a Policy Objective,” Federal Tax Policy for Economic Growth and Stability, Joint Committee on the Economic Report, Washington, D. C., November 9, 1955, p. 32.

  1. General Objectives and Policies

Keynes* J. M., “An Open Letter,” The New York Times, 1933, and Readings in Fiscal Policy

Lerner*, A. P., “Functional Finance and the Federal Debt,” Social Research, 1943, and Readings in Fiscal Policy. Also Chapter 24 in his Economics of Control, New York, 1944

Hart, A. G., “’Model-Building’ and Fiscal Policy,” American Economic Review, 1945, and Readings in Fiscal Policy

Committee for Economic Development, “Taxes and the Budget: A Program for Prosperity in a Free Economy,” Readings in Fiscal Policy, 1947

Colm* G., “The Government Budget and the Nation’s Economic Budget,” Public Finance, 1948, and Readings

Friedman*, M., “A Monetary and Fiscal Framework for Economic Stability,” American Economic Review, 1948, and Readings

National Planning Association, “Federal Expenditure and Revenue Policy for Economic Stability,” 1949, Readings

Bach*, G. L., “Monetary-Fiscal Policy Reconsidered,” Journal of Political Economy, October, 1949, and Readings

United Nations*, National and International Measures for Full Employment (report by a group of experts appointed by the Secretary-General), Lake Success, New York, December, 1949

Simons*, H. C., Federal Tax Reform, Chicago, 1950

Viner*, J., “Full Employment at Whatever Cost,” The Quarterly Journal of Economics, August, 1950

Samuelson*, P. A., “Principles and Rules in Modern Fiscal Policy; A Neoclassical Reformulation,” Money, Trade, and Economic Growth: in Honor of John H. Williams, New York, 1951

Millikan, M., ed., Income Stabilization for a Developing Democracy, Yale, 1953

Rolph, E.R., The Theory of Fiscal Economics, Berkeley and Los Angeles, 1954

U. S. Congress, Joint Committee on the Economic Report, Federal Tax Policy for Economic Growth and Stability, December, 1955, Hearings

American Economic Association* Readings in Fiscal Policy, Homewood, Illinois, 1955

National Bureau of Economic Research, Policies to Combat Depression, a conference of the Universities-National Bureau Committee for Economic Research, 1956

Council of Economic Advisers*, the latest Annual Report

  1. Institutional Factors

Bailey, S. K., Congress Makes a Law: the Story behind the Employment Act of 1946, New York, 1950

Bailey, S. K. and H. D. Samuel, Congress at Work, New York, 1952

Blough, R., The Federal Taxing Process, New York, 1952

Smithies*, A., The Budgetary Process in the United States, Committee for Economic Development, New York, 1955

  1. Tax Incidence

Musgrave*, R. A., et al, “Distribution of Tax Payments by Income Groups,” National Tax Journal, March, 1951

Little, I. M. D., “Direct versus Indirect Taxes, Economic Journal, September, 1951

Musgrave*, R. A., “On Incidence,” Journal of Political Economy, August, 1953

Bach*, G. L., “The Impact of Moderate Inflation on Income and Assets of Economic Groups,” Federal Tax Policy for Economic Growth and Stability, Joint Committee on the Economic Report, Washington, D. C., November 9, 1955, p. 71

Musgrave*, R.A., “Incidence of the Tax Structure and its Effects on Consumption,” Federal Tax Policy for Economic Growth and Stability, Joint Committee on the Economic Report, Washington, D. C., November 9, 1955, p. 96

  1. Cyclical Aspects

Slichter*, S. H., “The Economics of Public Works,” American Economics Review, 1934, and Readings in Fiscal Policy

Lutz, H. L., “Federal Depression Financing and its Consequences,” Harvard Business Review, 1938, and Readings

Myrdal* G., “Fiscal Policy in the Business Trade,” American Economic Review Supplement, 1939, and Readings

Hagen, E. E., “Timing and Administering Fiscal Policy,” American Economic Review, May, 1948

Committee on Public Issues of the American Economic Association*, “The Problem of Economic Instability,” American Economic Review, 1950, and Readings

Smithies*, A., “The American Economic Association Committee Report on Economic Instability,” American Economic Review, 1951, and Readings

Phillips, A. W., “Stabilization Policy in a Closed Economy,” The Economic Journal, June, 1954, pp. 290-323

Committee for Economic Development*, Problems in Anti-Recession Policy, September 1954

  1. Alternative Budgets for Full Employment

Kaldor*, N., Appendix C in W. H. Beveridge, Full Employment in a Free Society, 1945

Musgrave*, R. A., “Alternative Budget Policies for Full Employment,” American Economic Review, 1945, and Readings

Musgrave*, R. A. and M. H. Miller, “Built-In Flexibility,” American Economic Review, 1948 and Readings

Bishop*, R. L., “Alternative Expansionist Policies,” Income, Employment and Public Policy: Essays in Honor of Alvin H. Hansen, New York, 1948

Stein, H., “Budget Policy to Maintain Stability,” Problems in Anti-Recession Policy, Committee for Economic Development, September, 1954

Hagen*, E. E., “Federal Taxation and Economic Stabilization,” Federal Tax Policy for Economic Growth and Stability, Joint Committee on the Economic Report, November 9, 1955, pp. 58-70

Lusher, D. W., “The Stabilizing Effectiveness of Budget Flexibility,” Policies to Combat Depression, a conference of the Universities-National Bureau Committee for Economic Research, Princeton, 1956

  1. Balanced Budget Multiplier

Wallich*, H. C., “Income-Generating Effects of a Balanced Budget,” Quarterly Journal of Economics, November, 1944

Haavelmo*, T., “Multiplier Effects of a Balanced Budget,” Econometrica, 1945, and Readings

Haberler, G., “Multiplier Effects of a Balanced Budget,” Econometrica, April, 1946

Baumol, W. J. and M. H. Preston, “More on the Multiplier Effects of a Balanced Budget under Full Employment,” American Economic Review, March, 1955

  1. The National Debt

Studenski, P., “The Limits of Possible Debt Burdens—Federal, State, and Local,” American Economic Review, Supplement, 1937

Haley*, R. F., “The Federal Budget: Economic Consequences of Deficit Financing,” American Economic Review, 1941, and Readings

Williams*, H. H., “Deficit Spending,” American Economic Review, February, 1941 and Postwar Monetary Plans and other Essays, 1944

Ratchford, B. U., “The Burden of a Domestic Debt,” American Economic Review, 1942, and Readings

Williams, J. H., “The Implications of Fiscal Policy for Monetary Policy and the Banking System,” Proceedings of the American Economic Association, 1942, and Readings

Domar*, E. D., “The ‘Burden of the Debt’ and the National Income,” American Economic Review, 1944, and Readings

Simons*, H., “On Debt Policy,” Journal of Political Economy, 1944, and Readings

Seltzer, L. H., “Is a Rise in Interest Rates Desirable or Inevitable?” American Economic Review, 1945, and Readings

Wallich, H. C., “Debt Management as an Instrument of Economic Policy,” American Economic Review, June, 1946

Roosa, R. V., “Integrating Debt Management and Open Market Operations,” American Economic Review, 1952, and Readings

Burkhead*, J., “The Balanced Budget,” Quarterly Journal of Economic, 1954, and Readings

  1. Inflation and War Finance

Sprague, O. M. W., “Loans and Taxes in War Finance,” American Economic Review, Proceedings, 1917, and Readings

Keynes*, J. M., How to Pay for the War, London, 1940

Smithies*, A., “The Behavior of Money National Income under Inflationary Conditions,” Quarterly Journal of Economics, 1942, and Readings

Fellner*, W. J., “Postscript on War Inflation: A Lesson from World War II,” American Economic Review, 1947, and Readings

Fetter*, F., “The Economic Reports of the President and the Problem of Inflation,” Quarterly Journal of Economics, 1949, and Readings

Wald, H. P., “Fiscal Policy, Military Preparedness, and Postwar Inflation,” National Tax Journal, 1949, and Readings

Hart, A. G., Defense Without Inflation, New York, 1951

  1. Effect on Incentives: Incentive Taxation

Domar*, E. D., and R. A. Musgrave, “Proportional Income Taxation and Risk Taking,” Quarterly Journal of Economic, May, 1954

Butters, J. K., and J. Lintner, Effect of Federal Taxes on Growing Enterprises, Boston, 1945

Groves*, H. M., Postwar Taxation and Economic Progress, New York, 1946, Chapter 11

Shelton, J. P., and G. Ohlin, “A Swedish Tax Provision for Stabilizing Business Investment,” American Economic Review, June, 1952

Brown*, R. S., “Techniques for Influencing Private Investment,” Income Stabilization in a Developing Democracy, M. Millikan, ed., 1953, pp. 416-432

Domar*, E. D., “The Case for Accelerated Depreciation,” Quarterly Journal of Economics, February, 1953

Butters*, J. K., “Taxation, Incentives, and Financial Capacity,” American Economic Review, Supplement, 1954, and Readings

Brown, E. C., “The New Depreciation Policy under the Income Tax: An Economic Analysis,” National Tax Journal, March, 1955

Goode*, R., “Accelerated Depreciation Allowances as a Stimulus to Investment,” Quarterly Journal of Economics, May, 1955

Break*, G. F., “Effects of Taxation on Work Incentives,” Federal Tax Policy for Economic Growth and Stability, Joint Committee on the Economic Report, Washington, D. C., November 9, 1955, p. 192

Brown*, E. C., “Weaknesses of Accelerated Depreciation as an Investment Stimulus,” Federal Tax Policy for Economic Growth and Stability, Joint Committee on the Economic Report, Washington, D. C., November 9, 1955, p. 495

Butters*, J. K., “Effects of Taxation on the Investment Capacities and Policies of Individuals,” Federal Tax Policy for Economic Growth and Stability, Joint Committee on the Economic Report, Washington, D. C., November 9, 1955, p. 126

Greenewalt*, C. H., “Effect of High Tax Rates on Executive Incentive,” Federal Tax Policy for Economic Growth and Stability, Joint Committee on the Economic Report, Washington, D. C., November 9, 1955, p. 185

Long*, C. D., “Impact of Federal Income Tax on Labor Force Participation,” Federal Tax Policy for Economic Growth and Stability, Joint Committee on the Economic Report, Washington, D. C., November 9, 1955, p. 153

Kaldor*, N., “An Expenditure Tax,” London, 1955

  1. Particular Taxes

Simons, H., Personal Income Taxation, Chicago, 1938

Brown*, E. C., “Analysis of Consumption Taxes in Terms of the Theory of Income Determination,” American Economic Review, March, 1950

Goode*, R., Corporation Income Tax, New York, 1951

Royal Commission on the Taxation of Profits and Income, First Report, February, 1953; Second Report, April, 1954; Final Report, June, 1955

Due, J. F., “Economics of Commodity Taxation and the Present Excise Tax System,” Federal Tax Policy for Economic Growth and Stability, Joint Committee on the Economic Report, Washington, D. C., November 9, 1955, p. 547

Keith*, G., “Economic Impact of the Corporation Income Tax,” Federal Tax Policy for Economic Growth and Stability, Joint Committee on the Economic Report, Washington, D. C., November 9, 1955, p. 658

Goode, R., “The Corporate Income Tax in a Depression,” Policies to Combat Depression, a conference of the Universities-National Bureau Committee for Economic Research, 1956

Merriam, I. C., “Social Security Programs and Economic Stability,” Policies to Combat Depression

Pechman, J. A., “Yield of the Individual Income Tax During A Recession,” Policies to Combat Depression

  1. Inter-Governmental Fiscal Relations

Maxwell*, J. A., “Intergovernmental Fiscal Devices for Economic Stabilization,” Federal Tax Policy for Economic Growth and Stability, Joint Committee on the Economic Report, Washington, D. C., November 9, 1955, p. 807

Heer*, C., “Stabilizing State and Local Finance,” Policies to Combat Depression, 1956

U. S. Treasury Department, Committee on Inter-Governmental Fiscal Relations, Federal, State, and Local Government Fiscal Relations, 78th Congress, 1st Session, Senate Document No. 69, 1943

U. S. Bureau of the Census, Compendium of State Government Finances (an annual series)

Same source, Compendium of City Government Finances (an annual series)

Tax Institute, Federal-State-Local Tax Correlation (A Symposium), December, 1953

The Council of State Governments, Federal Grants-in-Aid, 1949

  1. Growth and Economic Development

Bernstein*, E. M. and I. G. Patel, “Inflation in Relation to Economic Development,” International Monetary Fund, Staff papers, II, 1951-52

United Nations*: Fiscal Division, “Taxation and Economic Development in Asian Countries,” Economic Bulletin for Asia and the Far East, Vol. IV, November, 1953

Gurley, J. A., “Fiscal Policy in a Growing Economy,” Journal of Political Economy, December, 1953

Papers and Proceedings of the Conference on Agricultural Taxation and Economic Development, H. P. Wald, and J. N. Froomkin, eds., Cambridge, Massachusetts, 1954 (Harvard University Law School, International Program in Taxation)

  1. Special Problems

Clark*, C., “Public Finance and Changes in the Value of Money,” The Economic Journal, December, 1945

Clark*, C., “The Danger Point in Taxes,” Harper’s Magazine, December, 1950

Goode*, R., “An Economic Limit on Taxes: Some Recent Discussion,” National Tax Journal, September, 1952

Caplan, B., “A Case Study: The 1948-1949 Recession,” Policies to Combat Depression, 1956

Fox, K. A., “The Contribution of Farm Price Support Programs to General Economic Stability,” Policies to Combat Depression, 1956

Gordon, R. A., “Types of Depressions and Programs to Combat Them,” Policies to Combat Depression

Grebler, L., “Housing Policies to Comat Depression,” Policies to Combat Depression

Johnson, D. G., “Stabilization of International Commodity Prices,” Policies to Combat Depression

Owen, W., “Self-Liquidating Public Works to Combat Depression,” Policies to Combat Depression

Source: Duke University, David M. Rubenstein Rare Book and Manuscript Library. Economists’ Papers Archive. Box 17, Folder “Fiscal and Monetary Policy”.

___________________________

FINAL EXAMINATION
14.472 Fiscal Policy
Monday, May 20, 1957

E. D. Domar

ANSWER ALL QUESTIONS. THE QUALITY OF YOUR REASONING IS THE MOST IMPORTANT PART OF YOUR ANSWERS.

  1. [35%] Compare and contrast monetary and fiscal policies as methods of achieving a steadily expanding economy (without inflation or depression). Include, but don’t limit yourself to, the following points:
      1. The theoretical foundation of each.
      2. Methods used.
      3. Effects on distribution of income and wealth.
      4. Social and political repercussions of each.
      5. The effectiveness and limitations of each.

Do they overlap? Can you work out a synthesis?

  1. [20%] “Government spending tends to be like a drug, in that it takes larger and larger doses to get results, and all the time debt and taxes get higher and higher.”
    Analyze this statement and comment as fully as you can. Compare the effect of government expenditures with that of private.
  2. [15%] “The best cure against inflation is increased production.”
    Analyze this statement and comment on it. Include in your comments the monetary and fiscal implications of this statement.
  3. [15%] What are the so-called “Built-in-Stabilizers?” Discuss fully and indicate how they operate in (a) depression and (b) inflation.
  4. [15%] “The purpose of taxation is never to raise money but to leave less in the hands of the taxpayer.”
    Comment fully and indicate the limitations of this statement. Can you identify the author? (No great penalty if you cannot.)

Source: Duke University, David M. Rubenstein Rare Book and Manuscript Library. Economists’ Papers Archive. Box 16, Folder “Examination. Public Finance and Fiscal Policy”.

Notes on Final Exam:

Question II comes from a review of Stuart Chase, Where’s the Money Coming From? Published in the Monthly Bulletin of the National City Bank of New York that I was fortunate to find inserted into the Congressional Record Volume 93—Part 4 (May 8, 1947, p. 4827);

Question III. Domar liked this question enough to have used it at least twice. See January 23, 1958 Exam at Johns Hopkins; January 26, 1966 at M.I.T.;

Question V. The sentence quoted comes from Abba Lerner’s The Economics of Control, p. 307.

___________________________

Image Source: Evsey D. Domar at the MIT Museum legacy website.

 

Categories
Exam Questions M.I.T. Suggested Reading Syllabus

M.I.T. First core graduate macroeconomics. Syllabus, readings, exams. Domar and Harris, 1967-68

 

 Four out of the five times that the first term of the macroeconomics sequence at M.I.T. (Theory of Income and Employment) was taught in the second half of the 1960’s, it was taught by Evsey Domar . Earlier posts with materials for Domar’s course include the reading list and final exam for 1960-61, reading list and exams for 1965-66 , the exams for 1968-69, and the course evaluations for 1967/68-1969/70.

Responsible for the course section in 1967-68 was the assistant professor John Rees Harris (b. 1934, d. 2018, 1967 Northwestern Ph.D. in economics) [copy of his c.v. archived 14 February 2019]. Here is link to a video lunchtime talk by Harris at the Boston University conference “Development that Works” (March 11, 2011). The picture is a screen-capture from the video.

______________________

M.I.T.
THE THEORY OF INCOME AND EMPLOYMENT
14.451
1967-68
[first session]

I. ADMINISTRATIVE QUESTIONS

    1. Course number, my and Harris’s name, our office numbers, office hours Tu 2:30-3:30.
    2. Sitting chart. No compulsory attendance.
    3. Reading list. First part only. Required and recommended or optional. Responsible for all required reading, but not for the details. I don’t know them myself. Lectures are the skeleton of the course. Reserve in Dewey. Inform me if some books are absent.
    4. The National Income problem. It is due….
    5. Midterm exam in November. Final exam.
    6. Other administrative problems?

II. THE PURPOSE AND NATURE OF THE COURSE

To fill in the gaps and bring everyone to a common denominator, without pulling anyone down. Hence, some will find it a bit boring. Attendance is not compulsory.

It is an introductory course. Almost everything will be discussed in other courses, except National Income, Index of Industrial Production, etc. Growth and fluctuations; monetary economics, consumption function, investment decisions, etc.

III. COMMENTS ON MACROECONOMICS

At the beginning, was a very hot subject—the most interesting part of economics. Two reasons: (1) it was new: (2) the greatest deficiency was in the macro area. Emphasis in those days was on full employment, not growth. Growth came in after the second world war.

The close connection between macro economics and governmental policies.

Three [sic] aspects:

(1) understanding of macro problems by economists

(2) persuading the public—easy in England, very difficult here.

(3) Forecasts of the future—improvement

(4) The effectiveness of methods—also part of forecasts.

On the whole macro-policy has been very successful, sometimes by design, sometimes by luck. The tax reduction of 1964 was the first one for fiscal policy specifically. Less fear of a deficit—witness the present situation. But the tax rise is still a test.

Next step—economic growth. First models—macro type with one kind of goods, and investment with capital coefficients. Still being used, but they don’t get us far.

Growth is to a considerable extent a micro-problem, or at least a mixture of the two. Much more difficult for the government to legislate. How does one improve efficiency? Evaluation of investment projects, of economic effects of education, etc.

Some exaggeration—but the traditional macro theory suffers from its own success.

 

PART I NATIONAL INCOME AND RELATED ITEMS

First—to state the objectives, such as welfare (whose?), capacity to produce (what?), national prestige, evaluation of policies, curiosity about growth, etc.

How to bring order out of the chaos? Which goods and services, which transactions are to be recorded?

Define the purpose of economic activity:

(1) Welfare of all people (or citizens) of a given area

(2) Welfare of some people only (slaves or relatives excluded). Weights?

(3) Welfare of animals? The old lady and her cat?

The definition of welfare may lead to a definition of activities to be included.

Special activities: warfare (Sparta), capital formation, police protection, etc.

Market vs. non-market goods. Imputed items.

 

Source: Duke University. David M. Rubenstein Rare Book and Manuscript Library. Economists’ Papers Archive. Evsey D. Domar Papers. Box 17, Folder “Macroeconomics. Theory of National Income and Employment”.

_________________________

THEORY OF INCOME AND EMPLOYMENT
14.451
Fall Term 1967-68

E.D. Domar
J.R. Harris

READING LIST

The purpose of this list is to suggest to the student the sources in which the more important topics of the course are discussed from several points of view. His objectives should be the understanding of these topics and not the memorization of opinions and details.

The “optional” reading has been included for those students who wish to pursue some of the subjects in greater detail. Some of the items on the optional list may be more effective in their exposition, at least for some individuals, than those on the required list.

There exists a good (if a bit obsolete) textbook on macroeconomics—Gardner Ackley, Macroeconomic Theory (The Macmillan Company, New York, 1961). Its knowledge is necessary but not sufficient for passing the course. While several copies are on reserve at Dewey, the acquisition of private copies is recommended.

Students may also find it convenient to acquire the following books: Readings in Macroeconomics edited by M.G. Mueller (which contains a number of relevant articles) and possible the three National Income volumes published by the U.S. Department of Commerce and listed in Section I.

I. NATIONAL INCOME AND RELATED ITEMS
(September 19 – October 12)

REQUIRED

Ackley, Chapters 1-4.

Kuznets, S., National Income and Its Composition, Vol. I (New York, 1941), Chap. 1.

National Income 1954 Edition, A Supplement to the Survey of Current Business, U.S. Department of Commerce (Washington, D.C., 1954), pp. 27-60, 153-58.

U.S. Income and Output, A Supplement to the Survey of Current Business, U.S. Department of Commerce (Washington, D. C., 1958), pp. 50-105.

The National Income and Product Accounts of the United States, 1929-1965. U.S. Department of Commerce (Washington, D.C., 1966). Browse through the statistics tables of the three volumes to find out what is available where.

Bergson, A. The Real National Income of Soviet Russia since 1928, Ch. 3 on “Methods and Procedures”, (Cambridge, Mass., 1961).

Griliches, Z. “Notes on the Measurement of Price and Quality Changes”, in Models of Income Determination, Studies in Income and Wealth, Vol. 28 by the Conference on Research in Income and Wealth, National Bureau of Economic Research, 1964, pp. 381-418.

Leontief, W. W., “Output, Employment, Consumption and Investment,” Quarterly Journal of Economics, Vol. 58 (February, 1944), pp. 290-314.

Leontief, Studies in the Structure of the American Economy (New York, 1953), pp. 27-35.

Dorfman, R., “The Nature and Significance of Input-Output,” Review of Economics and Statistics, Vol. 36 (May, 1954), pp. 121-33.

Domar, E. D., “On the Measurement of Technological Change,” The Economic Journal, Vol. 71 (December, 1961), pp. 709-29. [Read only pp. 709-14, 726-29.]

Board of Governors of the Federal Reserve System, Industrial Production 1959 Revision (Washington, 1960), pp. iii-41. [Look for the method, not for statistical details.]

Domar, E. D., “An Index-Number Tournament,” The Quarterly Journal of Economics, Vol. LXXXI (May, 1967), pp. 169-88.

Sigel, S. J., “A Comparison of the Structures of Three Social Accounting Systems,” National Bureau of Economic Research, Input-Output Analysis: An Appraisal, The Conference on Research in Income and Wealth, Studies in Income and Wealth, Vol. 18 (Princeton, 1955), pp. 253-89.

 

OPTIONAL READINGS:

Jaszi, G., “The Statistical Foundations of the GNP,” Review of Economics and Statistics, Vol. 38 (May, 1956), pp. 205-14.

Lewis, Wilfred, Jr., “The Federal Sector in National Income Models,” and comments by Hickman and Pechman, in Conference on Research in Income and Wealth, Models of Income Determination (Princeton, 1964), Vol. 28, pp. 233-78.

Bailey, M. J., National Income and the Price Level (New York, 1962), pp. 269-300.

Kuznets, S., National Income and Its Composition (New York, 1941).

Ruggles, R. and N., National Income Accounts and Income Analysis (New York, 1956).

Ruggles, “The U.S. National Accounts,” American Economic Review, Vol. 49, (March, 1959), pp. 85-95.

National Bureau of Economic Research, The National Economic Accounts of the United States, Review, Appraisal and Recommendations, General Series 64, (Washington, 1958).

Organization for European Economic Cooperation, A Standardised System of National Accounts, (Paris, 1952).

Gilbert, M. and I. B. Kravis, An International Comparison of National Products and the Purchasing Power of Currencies, A Study of the United States, the United Kingdom, France, Germany and Italy, Organization for European Economic Cooperation (Paris, 1954).

Gilbert, M., Comparative National Products and Price Levels, A Study of Western Europe and the United States, Organization of European Economic Cooperation, (Paris, 1958).

United Nations, Yearbook of National Accounts Statistics, the latest issue.

United Nations, National Income Statistics, the latest issue.

United Nations, World Economic Survey and other Economic Surveys.

Studenski, The Income of Nations. Theory, Measurement, and Analysis: Past and Present (New York, 1958). [A wealth of information, particularly of historical character.]

Nove, A., “The United States National Income A La Russe,” Economica, Vol. 23, 1956.

Bergson, A. The Real National Income of Soviet Russia Since 1928 (Cambridge, Massachusetts, 1961). (The rest of the book).

Kravis, I. B., “Relative Income Shares in Fact and Theory,” American Economic Review, Vol. 49 (December, 1959), pp. 917-49.

Samuelson, P. A., “Evaluation of Real National Income,” Oxford Economic Papers (New Series), 1950, pp. 1-29.

Samuelson, “The Evaluation of ‘Social Income’: Capital Formation and Wealth,” in F. A. Lutz and D. C. Hague, editors, The Theory of Capital (London, 1961).

Leontief, W. W., The Structure of American Economy (New York, 1941).

Leontief, Studies in the Structure of the American Economy (New York, 1953).

Taskier, C. E., Input-Output Bibliography 1955-1960, United Nations (New York, 1961).

Evans, W. D., and M. Hoffenberg, “The Interindustry Relations Study for 1947,” Review of Economics and Statistics, Vol. 34, (May, 1952), pp. 97-142.

Stewart, I. G., “The Practical Uses of Input-Output Analysis,” Scottish Journal of Political Economy, Vol. 5, (February, 1958).

Dosser, D. and A. T. Peacock, “Input-Output Analysis in an Under-Developed Country: A Case Study,” Review of Economic Studies, Vol. 25 (October, 1957).

Input-Output Analysis: An Appraisal, Studies in Income and Wealth by the Conference on research in Income and Wealth, Vol. 18 (Princeton, 1955).

Solow, R. M. “Technical Change and the Aggregate Production Function,” Review of Economics and Statistics, Vol. 39 (August, 1957), pp. 312-20.

Abramovitz, M., “Resources and Output in the United States Since 1870,” American Economic Review, Papers and Proceedings, Vol. 46 (May, 1956), pp. 5-23, reprinted as National Bureau of Economic Research, Occasional Paper 52 (New York, 1956).

Kendrick, J. W., Productivity Trends in the United States (Princeton, 1961).

Denison, E. F., Sources of Economic Growth in the United States and the Alternatives Before Us (New York, 1962).

Abramovitz, M., “Economic Growth in the United States,” American Economic Review, Vol. 52 (September, 1962), pp. 762-82. [This is a review of Denison’s Book.]

Moorsteen, R. H., “On Measuring Productive Potential and Relative Efficiency,” Quarterly Journal of Economics, Vol. 75 (August, 1961), pp. 451-67.

Fabricant, S., The Output of Manufacturing Industries, 1899-1937 (New York, 1940), particularly Chapter 1.

United Nations, Statistical Office, Index Numbers of Industrial Production, St/Stat/ Ser/ F1 (New York, 1950).

Board of Governors of the Federal Reserve System, Flow of Funds in the United States 1939-53 (Washington, D. C., 1955).

Powelson, J. P., National Income and Flow-Of-Funds Analysis (New York, 1960).

Measuring the Nation’s Wealth, National Bureau of Economic Research, Studies in Income and Wealth, Vol. 29 (Washington, D. C., 1964).

 

READING LIST—SECOND INSTALLMENT
II. GENERAL AGGREGATIVE SYSTEMS—FIRST APPROXIMATION
(October 17 – October 31).

REQUIRED:

Ackley, Parts II and III.

Keynes, J. M., The General Theory of Employment, Interest and Money (London and New York, 1936). [Omit the appendixes to Chapters 6 and 19.]

Note: Neither book is arranged in the order of this reading list. Hence these two assignments apply to other sections of it as well.

Wells, P., “Keynes’ Aggregate Supply Function: A Suggested Interpretation,” The Economic Journal, Vol. 70 (September, 1960), pp. 536-42.

Johnson, H. G. and the discussants, “The General Theory After Twenty-five Years,” American Economic Review Papers and Proceedings, Vol. 60 (May, 1961), pp. 1-25.

Klein, L. R., “The Empirical Foundations of Keynesian Economics,” in K. K. Kurihara, ed., Post Keynesian Economics(New Brunswick, N. J., 1954), pp. 277-319.

 

OPTIONAL READINGS:

Lekachman, Robert, Keynes’ General Theory: Reports of Three Decades, (New York and London, 1964).

Patinkin, D., Money, Interest, and Prices, Second Edition, (New York, 1965).

American Economic Association, Readings in Business Cycle Theory (Philadelphia, 1944), Essays 5, 7, 8.

American Economic Association, Readings in the Theory of Income Distribution (Philadelphia, 1946), Essay 24.

Metzler, “Three Lags in the Circular Flow of Income,” in Income, Employment and Public Policy, Essays in Honor of Alvin H. Hansen (New York, 1948), pp. 11-32.

Harris, S. E., The New Economics (New York, 1947), Essays 8-19, 31-33, 38-46.

Lerner, A. P., Economics of Control (New York, 1944), Chapters 21-23, 25.K

Kurihara, K. K., Post Keynesian Economics (New Brunswick, N. J., 1954).

Klein, L. R., The Keynesian Revolution, (New York, 1947), Chapters 3-5.

Ellis, H. S., A Survey of Contemporary Economics, Vol. 1, (Philadelphia, 1948), Chapter 2.

Burns, A. F., “Economic Research and the Keynesian Thinking of Our Times,” in his The Frontiers of Economic Knowledge, (Princeton, 1954), or in the Twenty-Sixth Annual Report of the National Bureau of Economic Research, Inc.(New York, 1946). See also the discussion by Hansen and Burns in the Review of Economic Statistics (November, 1947).

Dillard, D., “The Influence of Keynesian Economics on Contemporary Thought,” American Economic Review, Papers and Proceedings, 1957.

Hutt, W. H., Keynesianism: Retrospect and Prospect (Chicago, 1963).

Friedman, Milton, and G. S. Becker, “A Statistical Illusion on Judging Keynesian Models,” Journal of Political Economy, Vol. 55 (February, 1957), pp. 64-75.

 

III. PRICE FLEXIBILITY AND EMPLOYMENT
(November 2-9)

REQUIRED:

Patinkin, D., Money, Interest, and Prices, Second ed., (New York, 1965), Chapters 9-11.

Pigou, A. C., “The Classical Stationary State,” Economic Journal (December, 1943).

Power, J. H., “Price Expectations, Money Illusion and the Real Balance Effect,” Journal of Political Economy, Vol. 67 (April, 1959).

Mayer, T., “The Empirical Significance of the Real Balance Effect,” Quarterly Journal of Economics, Vol. 73 (May, 1959).

 

OPTIONAL READINGS:

Readings in Monetary Theory, Essay 13.

Schelling, T. C., “The Dynamics of Price Flexibility,” American Economic Review (September, 1949).

Lange, O., Price Flexibility and Employment (Bloomington, Indiana, 1944). [Get the main idea and omit the details.]

Friedman, M., “Lange on Price Flexibility and Employment,” American Economic Review (September, 1946).

Patinkin, D., Money, Interest, and Prices (Evanston, Illinois, 1956).

Hicks, J. R., “A Rehabilitation of ‘Classical Economics’,” Economic Journal, Vol. 47, (June, 1957).

 

IV. The Theory of Interest and the Demand for Money

Required:

Keynes, General Theory, Chapters 13-17.

Hansen, A., Monetary Theory and Fiscal Policy, Chapters 3,4.

Hicks, J. R., Value and Capital, Chapters 11, 12.

Friedman, M., “The Quantity Theory of Money—A Restatement,” Studies in the Quantity Theory of Money.

Patinkin, D., Money, Interest and Prices, 2nd ed., Chapters VIII, XV.

Tobin, J., “Liquidity Preference as Behavior Towards Risk,” The Review of Economic Studies, February 1958, pp. 65-86.

 

Optional:

American Economic Association, Readings in the Theory of Income Distribution (Philadelphia, 1946), Essays 22, 23, 26.

American Economic Association, Readings in Monetary Theory, (New York, 1951), Essays 6, 11, 15.

Friedman, M. and A. J. Schwartz, A Monetary History of the United States 1867-1960 (Princeton, 1963).

Gurley, J. G., and E. S. Shaw, “Financial Aspects of Economic Development,” AER, vol. 65, September 1955, pp. 515-38.

Gurley, J. G., and E. S. Shaw, Money in a Theory of Finance (Washington, 1960).

Hart, A. G., and P. B. Kenen, Money, Debt and Economic Activity, Third Ed., (Englewood Cliffs, N. J., 1961).

Lydall, H., “Income, Assets, and the Demand for Money,” Review of Economics and Statistics, vol. 40, February 1958, pp. 1-14.

Lutz, F. A., “The Interest Rate and Investment in a Dynamic Economy,” AER, December 1945).

Matthews, R. C. O., “Liquidity Preference and the Multiplier,” Economica, vol. 28, February 1961, pp. 37-52.

Patinkin, D., “Liquidity Preference and Loanable Funds: Stock and Flow Analysis,” Economica, Vol. 25, November 1958.

Review of Economics and Statistics Supplement, vol. 45, February 1963, on “The State of Monetary Economics.”

Wright, A. L., “The Rate of Interest in a Dynamic Model,” Quarterly Journal of Economics, vol. 72, August 1958, pp. 327-50.

 

Reading List—Third Installment
V. Consumption and Saving

Required:

Clower, R.W., “The Keynesian Counterrevolution: A Theoretical Appraisal,” in Hahn and Brechling (eds.), The Theory of Interest Rates (Macmillan, 1965).

Davidson, P., “A Keynesian View of Patinkin’s Theory of Employment,” E.J., September 1967.

Leijonhufvud, A., “Keynes and the Keynesians: A Suggested Interpretation,” AER, May 1967.

Ackley, Chapters 10, 11, 12.

Keynes, General Theory, Chapters 8, 9, 10.

Hagen, E.,”The Consumption Function: A Review Article,” Review of Economics and Statistics, XXXVII, Feb. 1955, pp. 48-54.

Duesenberry, J. S., Income, Saving, and the Theory of Consumer Behavior, Chapters 3, 4.

Friedman, M., A Theory of the Consumption Function, Chapters 1, 2, 3, 9.

Ando, A. and Modigliani, F., “The ‘Life Cycle’ Hypothesis of Saving,” AER, March 1963, pp. 55-85; March 1964, pp. 111—113.

Farrell, M. J., “The New Theories of the Consumption Function,” E.J., vol. 69, December, 1959, pp. 678-96.

Lintner, J., “The Determinants of Corporate Saving,” Savings in the Modern Economy (W. Heller, ed.), pp. 230-55.

Lintner, J. and discussants, “Distribution of Income of Corporations Among Dividends, Retained Earnings, and Taxes,” AER, vol. 46, May 1956, pp. 97-118.

Friend, I., and Kravis, I.B., “Entrepreneurial Income, Saving and Investment,” AER, vol. 47, June 1957, pp. 269-301.

Lubell, H., “Effects of Redistribution of Income on Consumers’ Expenditures,” AER, vol. 37, March 1947, pp. 157-170.

________, “A Correction,” AER, vol. 37, December 1947, p. 930.

Domar, E. D., Essays in the Theory of Economic Growth (New York, 1957), pp. 154-67, 195-201.

Bronfenbrenner, Yomana and Lee, “A Study in Redistribution and Consumption,” Review of Economics and Statistics, May 1955, pp. 149-59.

Tobin, J., “Asset Holdings and Spending Decisions,” AER May 1952, pp. 109-23.

Crockett, Jean, “Income and Asset Effects on Consumption: Aggregate and Cross Section,” and comments by D. B. Suits, in N.B.E.R., Models of Income Determination, pp. 97-136.

Tobin, J., “On the Predictive Value of Consumer Intentions and Attitudes,” The Review of Economics and Statistics, vol. 41, February 1959, pp. 1-11.

 

Optional

Bailey, M. J., “Saving and the Rate of Interest,” Journal of Political Economy, vol. 45, August 1957, pp. 279-305. Reprinted in Landmarks in Political Economy, edited by E. J. Hamilton, A. Rees, and H.G. Johnson (Chicago, 1962), pp. 583-622.

Brown, B., and F. M. Fisher, “Negro-White Savings Differentials and the Modigliani-Brumberg Hypothesis,” Review of Economics and Statistics, vol. 40, February 1958, pp. 79-81.

Brown, E. C., Solow, R. M., Ando, A., and J. Karekan, “Lags in Fiscal and Monetary Policy,” in Commission on Money and Credit, Stabilization Policies (Englewood Cliffs, 1963), pp. 1-165.

Clark, J.M., “Note on Income Redistribution and Investment,” AER, vol. 37, December 1947, p. 931.

Dennison, E. F., “A Note on Private Saving,” Review of Economics and Statistics, August 1958.

Dobrovolsky, S. P., Corporate Income Retention 1915-43 (New York, 1951). (Omit the details.)

Domar, E.D., Essays in the Theory of Economic Growth (New York 1957), pp. 154-67, 195-201.

Ferber, R., “The Accuracy of Aggregate Savings Functions in the Post-War Years,” Review of Economics and Statistics, vol. 37, May 1955, pp. 134-48.

Friedman, M., and G. Becker, “A Statistical Illusion in Judging Keynesian Models,” JPE, vol. 65, February 1957.

Friend, I., and S. Schor, “Who Saves?,” The Review of Economics and Statistics, vol. 41, May 1959, pp. 213-45.

Goldsmith, R. W., A Study of Saving in the United States, three volumes (Princeton, 1952).

Gordon, M. J., “The Optimum Dividend Rate,” presented at the sixth Annual International Meeting of the Institute of Management Sciences, Paris, September 1959. (On library reserve.)

Heller, W. W., Boddy, F. M., and C. L. Nelson, Savings in the Modern Economy, a Symposium (Minneapolis, 1953).

Katona, G., and E. Mueller, Consumer Expectations 1953-56 (Ann Arbor, Michigan, 1956).

Rees, and Johnson, H. G., (Chicago, 1962), pp. 583-622.

Klein, L. R., “The Friedman-Becker Illusion,” JPE, vol. 66, December 1958.

Klein, L. R., (ed.), Contributions of Survey Methods to Economics (New York, 1954).

Morgan, J. N., Consumer Economics (New York, 1955).

Modigliani, F., and R. Brumberg, “Utility Analysis and the Consumption Function: An Interpretation of Cross-Section Data,” in Kurihara, K. K., (ed.), Post Keynesian Economics (New Brunswick, N. J., 1954), pp. 388-436.

Mincer, J., “Employment and Consumption,” Review of Economics and Statistics, vol. 42, February 1960, pp. 20-26.

Zellner, Arnold, “The Short-Run Consumption Function,” Econometrica, (October, 1957).

 

VI. Investment

 

Required

Ackley, Chapter 17.

Keynes, General Theory, Chapters 11, 12.

White, W. H., “Interest Inelasticity of Investment Demand,” AER, vol. 46, September 1956, pp. 565-587.

Knox, “The Acceleration Principle and the Theory of Investment,” Economica, August 1952, pp. 269-97.

Meyer, J., and E. Kuh, The Investment Decision, Chapters 2, 8, 12.

Eisner, R., “Investment: Fact and Fancy,” Jorgenson, D.W., “Capital Theory and Investment Behavior,” Kuh, E., “Theory and Institutions in the Study of Investment Behavior,”: all three in AER, May 1963, pp. 237-268.

Lovell, M.C., “Determinants of Inventory Investment,” in N.B.E.R., Models of Income Determination, pp. 177-216.

Solomon, E., ed., The Management of Corporate Capital, pp. 48-55, 67-73.

Witte, J. G., “The Microfoundations of the Social Investment Function,” Journal of Political Economy, vol. 71, October 1963, pp. 441-56.

 

Optional

Andrews, P.W.S., “Further Inquiry into the Effects of Rates of Interest,” Oxford Economic Papers, February 1940, pp. 32-73.

Brockie, M.D., and A.L. Grey, “The Marginal Efficiency of Capital and Investment Programming,” Economic Journal, vol. 46, December 1956.

Cunningham, N.J., “Business Investment and the Marginal Cost of Funds,” Metroeconomica, vol. 10, August 1958.

Cunningham, N.J., “Business Investment and the Marginal Cost of Funds,” Part II, Metroeconomica, December 1958.

Duesenberry, J., Business Cycles and Economic Growth (New York, 1958), Chapters 4-7.

Ebersole, J.F., “The Influence of Interest Rates,” Harvard Business Review, vol. 17, 1938, pp. 35-39.

Foss, M.F., “Manufacturers’ Inventory and Sales Expectations—A Progress Report on a New Survey,” Survey of Current Business, August 1961.

Foss, M.F., and V. Natrella, “Ten Years’ Experience with Business Investment Anticipations,” Survey of Current Business, January 1957.

Foss, M.F., “Investment Plans and Realizations—Reasons for Differences in Individual Cases,” Survey of Current Business, June 1957.

Friend, I., and J. Bronfenbrenner, “Business Investment Programs and Their Realization,” Survey of Current Business, December 1950.

Grey, A.L., and M.D. Brockie, “The Rate of Interest, Marginal Efficiency of Capital and Net Investment Programming: A Rejoinder,” Economic Journal, June 1959.

Heller, W.W., “The Anatomy of Investment Decisions,” Harvard Business Review, March 1951, pp. 95-103.

Henderson, H.D., “The Significance of the Rate of Interest,” Oxford Economic Papers, October 1938, pp. 1-13.

Hirschleifer, J., “On the Theory of Optimal Investment Decision,” The Journal of Political Economy, vol. 66, August 1958, pp. 329-352. (An excellent but difficult paper.)

James, E., A Reconsideration of the Theoretical Criteria for Optimum Investment Planning (M.I.T. doctoral dissertation 1961).

Lerner, A.P., “On the Marginal Product of Capital and the Marginal Efficiency of Investment,” Journal of Political Economy, vol. 51, February 1953, pp. 1-14. Reprinted in Landmarks in Political Economy edited by E.J. Hamilton, A. Rees, and H.G. Johnson (Chicago, 1962), pp. 538-58.

Lovell, M.C., “Determinants of Inventory Investment,” in Conference on Research in Income and Wealth, Models of Income Determination (Princeton, 1964), vol. 28, pp. 177-232.

Lutz, F.A., and V., The Theory of Investment of the Firm (Princeton, 1951).

Lydall, H.F., “The Impact of the Credit Squeeze on Small and Medium Sized Manufacturing Firms,” Economic Journal, vol. 47, September 1957.

Meade, J.E., and P.W.S. Andrews, “Summary of Replies to Questions on Effects of Interest Rates,” and “Further Inquiry into the Effects of Rates of Interest,” Oxford Economic Papers, No. 1, 1938 and No. 3, 1940.

N.B.E.R., The Quality and Economic Significance of Anticipations Data, A Conference of the Universities—National Bureau Committee for Economic Research (Princeton, 1960).

Penrose, E.T., The Theory of the Growth of the Firm (Oxford, 1959).

Penrose, E.T., “Limits to the Growth and Size of Firms,” AER Papers and Proceedings, vol. 45, May 1955, pp. 531-43.

Pitchford, J.D. and A.J. Hagger, “A Note on the Marginal Efficiency of Capital,” Economic Journal, vol. 48, September 1958, pp. 597-600.

Robinson, J., The Accumulation of Capital (London, 1956). (Wish we had time for it.)

Sayers, R.S., “Business Men and the Terms of Borrowing,” Oxford Economic Papers, February 1940, pp. 23-31.

Spiro, A., “Empirical Research and the Rate of Interest,” Review of Economics and Statistics, vol. 40, February 1958.

Lintner, J., “Corporation Finance: Risk and Investment,” in N.B.E.R., Determinants of Investment Behavior (Robert Ferber editor), pp. 215-54.

Jorgenson, D.W., “The Theory of Investment Behavior,” in N.B.E.R., Determinants of Investment Behavior, pp. 129-55.

Miller, M.H. and F. Modigliani, “Estimates of the Cost of Capital Relevant for Investment Decisions under Uncertainty,” in N.B.E.R., Determinants of Investment Behavior, pp. 179-214.

Miller, M.H. and F. Modigliani, “Reply,” in N.B.E.R., Determinants of Investment Behavior, pp. 260-70.

Lovell, M.C., “Sales Anticipations, Planned Inventory Investment, and Realizations,” in N.B.E.R., Determinants of Investment Behavior, pp. 537-80.

 

Reading List—Fourth Installment
VII. Multiplier and Accelerator

Required

Kahn, R.F., “The Relation of Home Investment to Unemployment,” Economic Journal, 1931. Republished in Hansen and Clemence, Readings in Business Cycles and National Income (New York, 1953), Essay 15.

Readings in Business Cycle Theory, Essays 9-12.

Haavelmo, T., “Multiplier Effects of a Balanced Budget,” Econometrica, 1945, reprinted in Readings in Fiscal Policy, pp. 335-343.

Salant, William A., “Taxes, Income Determination, and the Balanced Budget Theorem,” The Review of Economics and Statistics, May 1957. Reprinted in Gordon and Klein (eds.) A.E.A. Readings in Business Cycles (1965).

Tsiang, S.C., “Accelerator, Theory of the Firm, and the Business Cycle,” Quarterly Journal of Economics, vol. 65, 1951.

 

Optional

Tinbergen, “Statistical Evidence on the Acceleration Principle,” Economica, vol. 5, 1938.

Eisner, R., “Capital Expenditures, Profits, and the Acceleration Principle,” and comments by G.H. Hickman, in Conference on Research in Income and Wealth, Models of Income Determination, (Princeton, 1964), vol. 28, pp. 137-176.

Peston, M.H., “Generalizing the Balanced Budget Multiplier,” and “Comment” by W.A. Salant, The Review of Economics and Statistics (August, 1958).

Bowen, W.G., “The Balanced-Budget Multiplier: A Suggestion for a More General Formulation,” The Review of Economics and Statistics, May 1957.

Goodwin, R.M., “The Multiplier” in Seymour E. Harris, ed., The New Economics (New York, 1947), pp. 482-99.

Chenery, H.B., “Overcapacity and the Acceleration Principle,” Econometrica, vol. 20, January 1952, pp. 1-28.

Caff, J.T., “A Generalization of the Multiplier-Accelerator Model,” The Economic Journal, vol. 69, March 1961, pp. 36-52.

Kuznets, S., “Relation Between Capital Goods and Finished Products in the Business Cycle,” in Economic Essays in Honor of Wesley Clair Mitchell, (New York, 1935).

Knox, A.D. “The Acceleration Principle and the Theory of Investment: A Survey,” Economica, vol. 19, 1952.

Harrod, R.F., Towards a Dynamic Economics (London, 1948).

Hicks, J.R., A Contribution to the Theory of the Trade Cycle (Oxford, 1950).

Goodwin, R.M., “Problems of Trend and Cycle,” Yorkshire Bulletin, vol. 5, August 1953.

Ott, A.E., “The Relation Between the Accelerator and the Capital Output Ratio,” Review of Economic Studies, vol. 25, June 1958.

Minsky, H., “Monetary Systems and Accelerator Models,” American Economic Review, vol. 47, 1957.

Friedman, M. and D. Meiselman, “The Relative Stability of Monetary Velocity and the Investment Multiplier in the United States, 1897-1958,” Stabilization Policies, Commission on Money and Credit (New Jersey, 1963), pp. 165-268.

Hester, D.D., “Keynes and the Quantity Theory: A Comment on the Friedman-Meiselman CMC Paper,” the reply by Friedman and Meiselman, and the rejoinder by Hester, The Review of Economics and Statistics, vol. XLVI, November 1964, pp. 364-377.

 

VIII. Employment and Inflation

Required

Ackley, Chap. XVI.

Bronfenbrenner, M. and F.D. Holzman, “Survey of Inflation Theory,” American Economic Review, LIII (Sept., 1963), pp. 593-661.

Higher Unemployment Rates, 1957-60, “Structural Transformation or Inadequate Demand,” Subcommittee on Economic Statistics of the Joint Economic Committee, Washington, 1961.

Hines, G.G., “Trade Unions and Wage Inflation in the United Kingdom,” R.E. Studies (October 1964).

Killingsworth, C.L., “Automation, Jobs and Manpower,” from Nation’s Manpower Revolution, Hearings before the Subcommittee on Employment and Manpower of the Committee on Labor and Public Welfare, 88th Congress, 1stsession, Washington, D.C., part 5, pp. 1461-1480.

Lipsey, Richard, “The Relation Between Unemployment and the Rate of Change in Money Wage Rates in the United Kingdom, 1862-1957: A Further Analysis,” Economica N.S. 27 (Feb. 1960). Reprinted in Klein and Gordon (eds.), Readings in Business Cycle Theory (1965).

Perry, George L., Unemployment, Money Wage Rates and Inflation (1966).

Phillips, “The Relation Between Unemployment and the Rate of Change of Money Wage Rates,” Economica (Nov., 1958), pp. 283-99.

Samuelson, P.A. and R. Solow, “Analytical Aspects of Anti-Inflation Policy,” American Economic Review (May 1960), pp. 177-94.

Solow, R.M., “The Case Against the Case Against the Guidelines,” in G. Schultz (ed.), Guidelines (1966).

 

Optional

Smithies, A., “The Behavior of Money National Income Under Inflationary Conditions,” Readings in Fiscal Policy, pp. 121-36.

Machlup, F., “Another View of Cost-Push and Demand-.Pull Inflation,” Review of Economics and Statistics, XLII, (May 1960), pp. 125-39.

Galbraith, J.K., “Market Structure and Stabilization Policy,” Review of Economics and Statistics (May 1957), pp. 124-33.

Hicks, J.R., “Economic Foundations of Wage Policy,” Economic Journal, (Sept. 1955), pp. 389-404.

Morton, W.A., “Trade Unionism, Full Employment and Inflation,” American Economic Review, (March 1950), pp. 13-39.

Slichter, S., “Do Wage-Fixing Agreements Have an Inflationary Bias,” American Economic Review, (May 1954), pp. 332-46.

Berman, B., “Alternative Measures of Structural Unemployment,” Employment Policy and the Labor Market, A.M. Ross, ed.

Joint Economic Committee, Higher Unemployment Rates, 1957-60, U.S. 87th Congress.

Galloway, “Labor Mobility, Resource Allocation and Structural Unemployment,” American Economic Review (Sept. 1963), pp. 694-716.

Gordon, R.A., “Has Structural Unemployment Worsened,” Industrial Relations (May 1964), pp. 53-77.

 

Source: Duke University. David M. Rubenstein Rare Book and Manuscript Library. Economists’ Papers Archive. Evsey D. Domar Papers. Box 15, Folder “Macroeconomics. Old Reading Lists”.

______________________

The Theory of Income and Employment
14.451
E. D. Domar [and] J. R. Harris

Midterm Examination
November 30, 1967

(One hour and fifteen minutes)

Please answer all questions. Use a separate book for each question.

  1. (25%) After the discovery that an hour of dancing a day increases a person’s efficiency, a hitherto unemployed dancing teacher was hired (to teach dancing to their employees or themselves) by the following units, one at a time;
    1. A beginning sculptor
    2. The Ford Foundation
    3. Sears, Roebuck & and Co.
    4. The Town of Concord
    5. The Head of the Mafia
    6. The Embassy of South Vietnam in Washington

Disregarding any indirect effects (such as the multiplier), indicate and explain how national income and product and the relevant subdivisions in money and in real terms are affected by this act on the assumption that (1) dancing is really effective, and (2) that it is not. Your reasoning is at least as important as your answer.

  1. (20%) “The Federal Reserve-type index is a poor numerator for the measurement of the Residual (Total Factor Productivity), or of any other productivity.”
    Comment fully.
  2. A visitor to M.I.T. has suggested recently that if the Federal Reserve Board buys bonds in the open market in periods of unemployment, then real output, prices and the interest rate—all three—will increase.
    Are these predictions consistent with those of Patinkin and Keynes? How would their predictions and your own results (you may or may not agree with those sages) be changed under conditions of full employment? Explain fully. (35%)
  3. (20%) A Russian economist once stated that Keynes’ variables were as follows:
Independent variables Dependent variables
1. Propensity to consume 1. Savings
2. Marginal efficiency of capital 2. Investment
3. Rate of interest 3. Level of employment
4. Liquidity preference

Comment. Be specific

Source: Duke University. David M. Rubenstein Rare Book and Manuscript Library. Economists’ Papers Archive. Evsey D. Domar Papers. Box 17, Folder “Macroeconomics. Examinations (1 of 3)”.

______________________

THE THEORY OF INCOME AND EMPLOYMENT
14.451
E. D. Domar [and] J. R. Harris

FINAL EXAMINATION
January 23, 1968

Three Hours

PLEASE ANSWER ALL QUESTIONS. THEY CARRY EQUAL WEIGHTS. USE A SEPARATE BOOK FOR EACH QUESTION.

  1. (A) National Product is defined by the U.S. Department of Commerce as the sum of all final goods (and services), each multiplied by its price.

(B) National Income is defined by it as the sum of all net incomes of certain recipients.

Discuss the following questions:

    1. What is a final good (or service) in (A)? What is the reason for this definition?
    2. What is the rationale for multiplying each good (or service) by its price? What assumptions are implied in this procedure? Are they realistic?
    3. Whose net incomes are aggregated? Why? What is a net income? What assumptions does this procedure imply? Are they realistic?
    4. Could you suggest changes or improvements in the above procedures? Justify them.

 

    1. “A high ratio of depreciation to investment is a sign of old age.”
    2. Why is a special definition of money required in the “Price Flexibility and Employment” problems? What is the definition? What assumptions does it rest on?
    3. “If the Balanced-Budget Multiplier is correct, isn’t Say’s Law also correct?

 

  1. Assume that this country is being threatened by inflation and discuss the pros and cons of the following measures allegedly directed against it. Whenever you can, indicate the positions which several economists whose theories were discussed in the course would take on these measures:
    1. (i) A temporary Federal sales tax on all goods and services, or
      (ii) a permanent tax of the same kind.
    2. (i) A redistribution of income from wages to profits, or
      (ii) a more equal distribution of income.
    3. Setting the rate of growth of labor productivity in each industry as the limit for the rate of increase of wages in that industry.
    4. (i) Remitting domestic taxes on American exports, or
      (ii) a reduction in import duties.
    5. A tax on all capital goods.

 

    1. Define and discuss the applicability to investment decisions of the marginal efficiency of investment (also called marginal efficiency of capital, or the internal rate of return) and the discounted present value. Can they give different ranking of investment projects? Why? Which measure would you use?
    2. What major modifications of investment criteria would be required if the investment was done by the U.S. Government in times of unemployment?
    3. Same, if the investment was done by the government of some underdeveloped country?

 

  1. Attempts to estimate the parameters of an aggregate consumption function for the U.S. have yielded the following results:
    1. Cross-section and short-term series analyses estimate a marginal propensity to consume somewhere in the range of .55-.70, this magnitude being lower than the average propensity to consume.
    2. Long-run time series analyses estimate a marginal propensity to consume equal to the average propensity of about .88.

Compare and contrast the assumptions, rationale and implications of the “Previous Peak Income”, “Permanent Income”, and “Lifetime Cycle” hypotheses, each of which purports to reconcile the above observations.

 

Source: Duke University. David M. Rubenstein Rare Book and Manuscript Library. Economists’ Papers Archive. Evsey D. Domar Papers. Box 17, Folder “Macroeconomics. Final Exams (2 of 3)”.

Image Source: Evsey D. Domar at the MIT Museum legacy website.

Categories
M.I.T. Teaching

M.I.T. Student survey regarding Domar’s core macro theory course, 1960

 

The previous post provided the course syllabus and reading list for the core graduate macroeconomics course taught at M.I.T. by Evsey Domar during the first term of the 1960-61 academic year. He took the job seriously enough to try surveying his students to gauge his pedagogic success. I post a transcription of the mimeographed survey questions that were distributed to the students and have inserted totals from a handwritten summary of results. The strongest signals to come out of this exercise were (i) that mixing graduate economics with graduate business students is probably unwise and (ii) that a heavy dose of national income and product accounting is bitter medicine to new graduate students.

An earlier post  presents the results of a survey of Domar’s course for several later cohorts (1967-69).

______________________

QUESTIONNAIRE ON THE THEORY OF NATIONAL INCOME AND EMPLOYMENT (14.451)
E. D. Domar
Fall term 1960-61

Because the course is given to students of widely different backgrounds I am not sure whether it is taught on the proper level and in the proper manner. The size of the class precludes personal interviews on the subject. You can do a great service to future students and to myself by filling out this questionnaire in the most thoughtful and honest manner.

  1. Do not write your name, but give the following information:
    1. Your major (such as Course XIV, XV, etc.)
    2. Concentration within your major, if you know it.
    3. Year of graduate work
    4. Was your undergraduate training in the U.S. or Canada? (Yes or No)
  2. The general level of the course was (check one):
too elementary about right too advanced
The course XIV: 1
XV: 0
XIV: 17
XV: 7
XIV: 0
XV: 1
I. National Income and Related Items XIV: 4
XV: 1
XIV: 19
XV: 12
XIV: 1
XV: 1
II. General Aggregative System XIV: 4
XV: 0
XIV: 17
XV: 9
XIV: 1
XV: 5
III. Theory of Interest XIV: 4
XV: 0
XIV: 17
XV: 9
XIV: 1
XV: 4
IV. Consumption Function XIV: 2
XV: 0
XIV: 20
XV: 10
XIV: 0
XV: 4
V. Multiplier and Accelerator XIV: 3
XV: 0
XIV: 16
XV: 8
XIV: 3
XV: 6
VI. Investment Decisions XIV: 3
XV: 0
XIV: 17
XV: 12
XIV: 2
XV: 2
VII. Price Flexibility XIV: 1
XV: 0
XIV: 16
XV: 8
XIV: 0
XV: 2
[column totals] XIV: 22
XV: 1
XIV: 139
XV: 75
XIV: 8
XV: 25
  1. Mathematics was used in the course (encircle one) not enough [XIV, 6; XV, 3], about right [XIV, 15; XV, 8], too much [XIV, 3; XV, 3].Specific examples, if any.[For course XIV the course is a bit too elementary (totals, 22:8). For course XV the course is too advanced (totals, 1:25). For all students, it is about right.
    Math should be used somewhat more.
    The scope is a bit too broad.
    So, the course should be made tighter, more advanced, and a bit more mathematical, particularly without course XV students.]
  2. The scope of the course was (encircle one) too narrow [XIV, 0; XV, 0], about right [XIV, 13; XV, 10], too broad [XIV, 3; XV, 3].Topics to be added are:
    Topics to be given greater attention are:
    Topics to be condensed are:
    Topics to be eliminated are:
    Other suggestions regarding the scope of the course
Expand Condense Eliminate
I. National Income and Related Items XIV: 1
XV: 1
XIV: 13
XV: 3
XIV: 1
XV: 0
II. General Aggregative System XIV: 8
XV: 5
XIV: 0
XV: 0
XIV: 0
XV: 0
III. Theory of Interest XIV: 10
XV: 1
XIV: 3
XV: 0
XIV: 0
XV: 0
IV. Consumption Function XIV: 2
XV: 0
XIV: 1
XV: 1
XIV: 0
XV: 0
V. Multiplier and Accelerator XIV: 2
XV: 1
XIV: 4
XV: 2
XIV: 0
XV: 0
VI. Investment Decisions XIV: 5
XV:2
XIV: 2
XV: 1
XIV: 0
XV: 0
VII. Price Flexibility XIV: 6
XV: 3
XIV: 2
XV: 0
XIV: 0
XV: 0
  1. Required reading material was (encircle one) too broad [XIV, 9; XV, 9], about right [XIV, 12; XV, 2], too concentrated [XIV, 0; XV, 1].
    Would you prefer a smaller number of required readings but with a more intensive study of each? [Yes: XIV, 13; XV, 10], [No: XIV, 5; XV, 1] Comment.Should the readings be discussed in class more thoroughly and often?
    [Yes: XIV, 7; XV, 6], [No: XIV, 7; XV, 5]How adequately did the Reserve shelf serve your needs? [Enough: XIV, 1; XV, 0], [Not enough: XIV, 3; XV, 2] Did you have difficulties in obtaining the readings?Suggestions for improvement[Course XIV would keep the breadth of the reading as now, possibly somewhat narrower. But XV definitely wants more narrow.
    Both would prefer a smaller number of readings, particularly XV
    Evenly divided on discussing readings in class.]
  2. Because of the size of the class, lectures usually took the place of discussions. Would you prefer more discussions? [Yes: XIV, 5; XV, 5], [No: XIV, 13; XV, 7]Would a smaller class materially improve the course? [Yes: XIV, 11; XV, 9], [No: XIV, 5; XV, 3]Any other suggestions?[No great demand for more discussion in the large class, but a clear demand for a smaller class.]
  3. Grade the instruction in the course (A, B, C and F) for the following qualities (as compared with other courses at M.I.T. and elsewhere).Clarity of exposition
    Intellectual stimulation
    Usefulness of information
    Enjoyment of the course
    General performance
Grades A+ A A/B B B/C C Below
C
Clarity XIV: 18
XV: 8
XIV: 0
XV: 1
XIV: 4
XV: 3
XIV: 0
XV: 1
XIV: 0
XV: 1
Stimulation XIV: 15
XV: 7
XIV: 0
XV: 2
XIV: 4
XV: 4
XIV: 2
XV: 0
XIV: 0
XV: 1
Usefulness XIV: 9
XV: 4
XIV: 1
XV: 1
XIV: 7
XV: 6
XIV: 1
XV: 2
XIV: 0
XV: 1
Enjoyment XIV: 1
XV: 0
XIV: 16
XV: 5
XIV: 0
XV: 1
XIV: 3
XV: 6
XIV: 2
XV: 1
XIV: 0
XV: 1
General XIV: 12
XV: 7
XIV: 3
XV: 2
XIV: 5
XV: 2
XIV: 0
XV: 2
XIV: 0
XV: 1

[Course XIV feels much better about the course than does XV. Clarity, stimulation and enjoyment get high marks. Usefulness—much less.]

  1. As you know, a third of the final examination is devoted to a substitute for a term paper. Would you prefer a usual term paper instead? [Yes: XIV, 11; XV, 2], [No: XIV, 10; XV, 7] Neither?
    [Existing exam—Yes: XIV, 5; XV, 7], [No: XIV, 4; XV, 2]
    [More exams—Yes: XIV, 5; XV, 1], [No: XIV, 0; XV, 0]

Comment on this.

[Course XIV prefer (slightly) a term paper and a midterm exam. Course XV don’t want a term paper.]

  1. Any other comments, suggestions, complaints, wishes, etc.[Useful comments
    Require Patinkin. Course XV find it too hard.
    Suggest the more important readings.
    People in the back couldn’t hear well.
    No seating assignment
    A midterm exam is frequently asked, otherwise, they don’t work on this course hard enough.
    A time schedule of the course.
    Better reading list.]

 

Source: Duke University. David M. Rubenstein Rare Book and Manuscript Library. Economists’ Papers Archive. Papers of Evsey D. Domar, Box 17, Folder “Macroeconomics. Questionnaire on the Theory of national Income & Employment (14.451)”.

Image Source: The M.I.T. mascot beaver on the cover of its yearbook, Technique 1949.

Categories
Exam Questions M.I.T. Suggested Reading Syllabus

M.I.T. National Income and Employment Theory. Readings and Final Exam. Domar, 1960-61

 

 

For this post I have transcribed Evsey Domar’s graduate core macroeconomics course outline/reading list along with the questions for the final examination from the first term of the 1960-61 academic year at M.I.T. Students from both course XIV (economics) and XV (management) took this course.

Note: Evsey Domar distributed a questionnaire to the students to obtain feedback on his course.  The next post provides the results from that survey. It is fairly apparent that Domar did not cover the last topic on the course reading list (economic growth).

Final exam grade distribution (50 exams)

A 16%
A- 12%
B+ 10%
B 20%
B- 14%
C 18%
D 8%
F 2%

Fun Fact. Among the students enrolled in the course and who took the final examination: Michael D. Intriligator, Peter A. Diamond, Ann Fetter Friedlaender, and Stephen Goldfeld.

The much expanded course reading list/bibliography and  both the midterm and final examinations from the 1965-66 academic year have been posted earlier.

_________________________

MASSACHUSETTS INSTITUTE OF TECHNOLOGY

THEORY OF NATIONAL INCOME AND EMPLOYMENT
14.451 Reading List
E. D. Domar Fall Term 1960-61

The purpose of this list is to suggest to the student the sources in which the more important topics of the course are discussed from several points of view. His objectives should be the understanding of these topics and not the memorization of opinions and details.

Items marked with an * are strongly recommended. (I don’t like to use the expression “required” in a graduate reading list.)

No term paper will be required, but each student is expected, in addition to his general reading, to choose one of the eight major divisions of the course (except that Part VIII should not be taken without prior consultation with the instructor) as a field of concentration. A part of the final examination will be designed to test his broader knowledge of the chosen field.

 

I. NATIONAL INCOME AND RELATED ITEMS

*Kuznets, S., National Income and Its Composition, (New York, 1941), particularly Vol. 1, Chapter 1

*Jaszi, G., “The Statistical Foundations of the GNP,” Review of Economics and Statistics, Vol. 38, 1956)
Ruggles, R. and N., National Income Accounts and Income Analysis (New York, 1956)

*U.S. Department of Commerce, U. S. Income and Output, A Supplement to the Survey of Current Business, 1958

*National Bureau of Economic Research, The National Economic Accounts of the United States, Review, Appraisal and Recommendations, General Series 64, Washington, 1958

Ruggles, “The U.S. National Accounts,” American Economic Review, March, 1959
Organization for European Economic Co-operation, A Standardised System of National Accounts, Paris, 1952

Gilbert, M. and I. B. Kravis, An International Comparison of National Products and the Purchasing Power of Currencies, A Study of the United States, the United Kingdom, France, Germany and Italy, Organization for European Economic Cooperation, Paris, 1954

Nove, A., “The United States National Income A La Russe,” Economica, Vol. 23, 1956

Gilbert, M., Comparative National Products and Price Levels, A Study of Western Europe and the United States, Organization of European Economic Cooperation, Paris, 1958

*Leontief, W. W., “Output, Employment, Consumption and Investment,” Quarterly Journal of Economics, Feb., 1944

Leontief, W. W. The Structure of American Economy (New York, 1951)

*Dorfman, R., “The Nature and Significance of Input-Output,” Review of Economics and Statistics, Vol. 36, 1954

Stewart, I. G., “The Practical Uses of Input-Output Analysis,” Scottish Journal of Political Economy, Vol. 5, (Feb. 1958)

Dosser, D. and A. T. Peacock, “Input-Output Analysis in an Under-Developed Country: A Case Study,” Review of Economic Studies, Vol. 25, Oct. 1957

*Sigel, S. J., “A Comparison of the Structures of Three Social Accounting Systems,” National Bureau of Economic Research, Input-Output Analysis: An Appraisal, The Conference on Research in Income and Wealth, Studies in Income and Wealth, Vol. 18, pp. 253-89

Board of Governors of the Federal Reserve System, Flow of Funds in the United States 1939-53 (Washington, D. C., 1955)

 

II. GENERAL AGGREGATIVE SYSTEM

Students without prior training in this field are advised to study D. Dillard, The Economics of John Maynard Keynes (New York, 1948), A. H. Hansen, A Guide to Keynes (New York, 1953), or K. Kurihara, Introduction to Keynesian Dynamics (New York, 1956).

*Keynes, J. M., The General Theory of Employment, Interest and Money (New York, 1936)

*American Economic Association, Readings in Business Cycle Theory (Philadelphia, 1944), Essays 5, 7

Harris, S. E., The New Economics (New York, 1947), essays 8-19, 31-33, 38-46.

*Lerner, A. P., Economics of Control (New York, 1944), chapters 21-23, 25

*Kurihara, K. K., Post Keynesian Economics (New Brunswick, N. J., 1954), essays 1, 11*

*American Economic Association, Readings in the Theory of Income Distribution (Philadelphia, 1946), essay 24

Klein, L. R., The Keynesian Revolution, (New York, 1947), chapters 3-5.

Ellis, H. S., A Survey of Contemporary Economics (Philadelphia, 1948), Vol. 1, chapter 2

*Income, Employment and Public Policy, Essays in Honor of Alvin H. Hansen (New York, 1948), essay I

*Burns, A. F., “Economic Research and the Keynesian Thinking of Our Times,” in his The Frontiers of Economic Knowledge, (Princeton, 1954), or in the Twenty-Sixth Annual Report of the National Bureau of Economic Research, Inc. (New York, 1946). See also the discussion by Hansen and Burns in the Review of Economic Statistics, November, 1947

Dillard, D., “The Influence of Keynesian Economics on Contemporary Thought,” American Economic Review, Papers and Proceedings, 1957

Patinkin, D., Money, Interest, and Prices (Evanston, Ill., 1956).

 

III. THEORY OF INTEREST

Readings in the Theory of Income Distribution, essays 22, 23, 26

*Hicks, J. R., Value and Capital (Oxford, 1957), Chapters 11-12

Readings in Monetary Theory, essays 6, 11, 15

*Gurley, J. G., and E. S. Shaw, “Financial Aspects of Economic Development,” American Economic Review, September, 1955)

Hart, A. G., Money, Debt and Economic Activity, Second Ed., (New York, 1953)

Patinkin, D., “Liquidity Preference and Loanable Funds: Stock and Flow Analysis,” Economica, Vol. 25, November, 1958

Patinkin, D., Money, Interest, and Prices (Evanston, Ill., 1956).

*Lydall, H., “Income, Assets, and the Demand for Money,” Review of Economics and Statistics, Vol. 40, Feb. 1958

Lutz, F. A., “The Interest Rate and Investment in a Dynamic Economy,” American Economic Review, Dec. 1945

See also Section VI — INVESTMENT DECISIONS

 

IV. CONSUMPTION FUNCTION

*Duesenberry, J. S., Income, Saving, and the Theory of Consumer Behavior (Cambridge, Massachusetts, 1949)

Haley, B. F., A Survey of Contemporary Economics (Homewood, Illinois, 1952), Vol. II, essay 2

Davis, T. E., “The Consumption Function as a Tool of Prediction,” The Review of Economics and Statistics, August 1952

Heller, W. W., Boddy, F. M., and C. L. Nelson, Savings in the Modern Economy, a Symposium (Minneapolis, 1953)

*Friend, I., and S. Schor, “Who Saves?,” The Review of Economics and Statistics, Vol. 41, May, 1959, Part 2

*Friend, I., and I. B. Kravis, “Entrepreneurial Income, Saving and Investment,”American Economic Review, June, 1957, pp. 269-301

Zellner, Arnold, “The Short-Run Consumption Function,” Econometrica, (Oct. 1957

*Ferber, R., “The Accuracy of Aggregate Savings Functions in the Post-War Years,” Review of Economics and Statistics, Vol. 37, May, 1955

*Tobin, J., “On the Predictive Value of Consumer Intentions and Attitudes,” The Review of Economics and Statistics, Vol. 41, Feb., 1959

Dennison, E. F., “A Note on Private Saving,” Review of Economics and Statistics, August, 1958
Post-Keynesian Economics, essay 15

Friedman, M., A Theory of the Consumption Function (Princeton, N. J., 1957)

Friedman, M., and G. Becker, “A Statistical Illusion in Judging Keynesian Models,” Journal of Political Economy, Vol. 65, Feb., 1957

Klein, L. R., “The Friedman-Becker Illusion,” Journal of Political Economy, Vol. 66, Dec., 1958

Morgan, J. N., Consumer Economics (New York, 1955)

Katona, G., and E. Mueller, Consumer Expectations 1953-56 (Ann Arbor, Michigan, 1956)

Klein, L. R., ed., Contributions of Survey Methods to Economics (New York, 1954)

 

V. MULTIPLIER AND ACCELERATOR

*Kahn, R. F., “The Relation of Home Investment to Unemployment,” Economic Journal, 1931. Republished in Hansen and Clemence, Readings in Business Cycles and National Income (New York, 1953), essay 15

*Readings in Business Cycle Theory, essays 11-12

*Haavelmo, T., “Multiplier Effects of a Balanced Budget,” Econometrica, 1945; reprinted in Readings in Fiscal Policy, pp. 335-343

*Salant, William A., “Taxes, Income Determination, and the Balanced Budget Theorem,” The Review of Economics and Statistics, May, 1957

Peston, M. H., “Generalizing the Balanced Budget Multiplier,” and “Comment” by W. A. Salant, The Review of Economics and Statistics, August, 1958

Bowen, W. G., “The Balanced-Budget Multiplier: A Suggestion for a More General Formulation,” The Review of Economics and Statistics, May, 1957

*Kuznets, S., “Relation Between Capital Goods and Finished Products in the Business Cycle,” in Economic Essays in Honor of Wesley Clair Mitchell, (New York, 1935)

*Knox, A. D. “The Acceleration Principle and the Theory of Investment: A Survey,” Economica, Vol. 19, 1952

*Tsiang, S. C., “Accelerator, Theory of the Firm, and the Business Cycle,” Quarterly Journal of Economics, Vol. 65, 1951

*Tinbergen, “Statistical Evidence on the Acceleration Principle,” Economica, Vol. 5, 1938

Harrod, R. F., Towards a Dynamic Economics (London, 1948)

Hicks, J. R., A Contribution to the Theory of the Trade Cycle (Oxford, 1950)

Goodwin, R. M., “Problems of Trend and Cycle,” Yorkshire Bulletin, Vol. 5, August, 1953

Ott, A. E., “The Relation Between the Accelerator and the Capital Output Ratio,” Review of Economic Studies, Vol. 25, June, 1958

Minsky, H., “Monetary Systems and Accelerator Models,” American Economic Review, Vol. 47, 1957

See also VI — INVESTMENT DECISIONS.

 

VI. INVESTMENT DECISIONS

Lutz, F. A., and V., The Theory of Investment of the Firm (Princeton, 1951)

*Heller, W. W., “The Anatomy of Investment Decisions,” Harvard Business Review, March, 1951, pp. 95-103

*Pitchford, J. D. and A. J. Hagger, “A Note on the Marginal Efficiency of Capital,” The Economic Journal, Vol. 48, 1958

*Meade, J. E., and P. W. S. Andrews, “Summary of Replies to Questions on Effects of Interest Rates,” and “Further Inquiry into the Effects of Rates of Interest,” Oxford Economic Papers, No. 1, 1938 and No. 3, 1940

*Ebersole, J. F., “The Influence of Interest Rates,” Harvard Business Review, Vol. 17, 1938, pp. 35-39

*Henderson, H. D., “The Significance of the Rate of Interest,” Oxford Economic Papers, October, 1938, pp. 1-13

Andrews, P. W. S., “Further Inquiry into the Effects of Rates of Interest,” Oxford Economic Papers, Feb., 1940, pp. 32-73

Sayers, R. S., “Business Men and the Terms of Borrowing,” Oxford Economic Papers, Feb., 1940, pp. 23-31

*White, W. H., “Interest Inelasticity of Investment Demand—The Case from Business Attitude Surveys Re-examined,” American Economic Review, Sept. 1956, pp. 565-587

Brockie, M. D., and A. L. Gray, “The Marginal Efficiency of Capital and Investment Programming,” Economic Journal, Vol. 46, December, 1956

White, W. H., “The Rate of Interest, the Marginal Efficiency of Capital, and Investment Programming,” Economic Journal, Vol. 48, March, 1958

Grey, A. L., and M. D. Brockie, “The Rate of Interest, Marginal Efficiency of Capital and Net Investment Programming: A Rejoinder,” Economic Journal, June, 1959

Spiro, A., “Empirical Research and the Rate of Interest,” Review of Economics and Statistics, Vol. 40 (February, 1958).

*Duesenberry, J., Business Cycles and Economic Growth (New York, 1958), Chapters 1-8

Meyer, John R., and Edwin Kuh, The Investment Decision (Cambridge, Mass., 1957)

Cunningham, N. J., “Business Investment and the Marginal Cost of Funds,” Metroeconomica, Vol. 10, August, 1958

Cunningham, N. J., “Business Investment and the Marginal Cost of Funds,” Part II, Metroeconomica, Dec., 1958

Wilson, T., “Cyclical and Autonomous Inducements to Invest,” Oxford Economic Papers, Vol. 5, 1953

Hirschleifer, J., “On the Theory of Optimal Investment Decision,” The Journal of Political Economy, Vol. 66, Aug., 1958

Lydall, H. F., “The Impact of the Credit Squeeze on Small and Medium Sized Manufacturing Firms,” Economic Journal, Vol. 47, Sept., 1957

*Penrose, E., “Limits to the Growth and Size of Firms,” American Economic Review Papers and Proceedings, May 1955, pp. 531-43

Friend, I., and J. Bronfenbrenner, “Business Investment Programs and Their Realization,” Survey of Current Business, December, 1950

*Foss, M. F., and V. Natrella, “Ten Years’ Experience with Business Investment Anticipations,” Survey of Current Business, January, 1957

*Foss, M. F., and V. Natrella, “Investment Plans and Realizations—Reasons for Differences in Individual Cases,” Survey of Current Business, June, 1957

See also III—THEORY OF INTEREST and V—MULTIPLIER AND ACCELERATOR

 

VII. PRICE FLEXIBILITY AND EMPLOYMENT

*Pigou, A. C., “The Classical Stationary State,” Economic Journal, Dec., 1943

*Lange, O., Price Flexibility and Employment (Bloomington, Indiana, 1944)

*Friedman, M., “Lange on Price Flexibility and Employment,” American Economic Review, Sept., 1946

Readings in Monetary Theory, Essay 13

Schelling, T. C., “The Dynamics of Price Flexibility,” American Economic Review, Sept. 1949

Patinkin, D., Money, Interest, and Prices (Evanston, Illinois, 1956)

Hicks, J. R., “A Rehabilitation of ‘Classical Economics’,” Economic Journal, Vol. 47, June, 1957

*Power, J. H., “Price Expectations, Money Illusion and the Real Balance Effect,” Journal of Political Economy, Vol. 67, April, 1959

*Mayer, T., “The Empirical Significance of the Real Balance Effect,” Quarterly Journal of Economics, Vol. 73, May, 1959

 

VIII. THEORY OF GROWTH

*Domar, E. D., Essays in the Theory of Economic Growth (New York, 1957), Foreword, Essays I, III-V

Fellner, W., Trends and Cycles I Economic Activity, (New York,1956)

Hansen, A. H., Fiscal Policy and Business Cycles (New York, 1941)

*Harrod, R. F., Towards a Dynamic Economics (London, 1948), Part III

Leontief, W. W., Studies in the Structure of the American Economy, (New York, 1953)

Robinson, J., The Accumulation of Capital, (London, 1956)

*Kuznets, Simon, “Towards a Theory of Economic Growth,” R. Leckachman, ed., National Policy for Economic Welfare at Home and Abroad, (New York, 1955)

*Solow, R. M., “A Contribution to the Theory of Economic Growth,” Quarterly Journal of Economics, Feb. 1956, pp. 65-94

*Solow, R. M., “Technical Change and the Aggregate Production Function,” Review of Economics and Statistics, August, 1957, pp. 312-320

 

Source:  Duke University, David M. Rubenstein Library. Economists’ Papers Archives. Papers of Evsey D. Domar, Box 15, Folder “Macroeconomics, Old Reading Lists”.

 

_________________________

Economics 14-451
E. D. Domar

FINAL EXAMINATION—Three Hours
January 24, 1961

Please use a separate book for each question.

 

Part I—One Hour

Write an essay in the field of your concentration as instructed in class. Please be specific.

 

Part II—Two Hours

Answer the THREE questions which are furthest removed from the topic discussed in Part I. They carry equal weights.

  1. “Thus the rate of interest is what it is because it is expected to become other than it is: if it is not expected to become other than it is, there is nothing left to tell us what it is…”
    1. Can you identify the author of this famous statement?
    2. Can you recognize whose interest theory he referred to?
    3. Explain and evaluate that theory critically.
    4. Present your own (original or otherwise) theory of interest.
  2. Write an essay on the subject of “The treatment of intermediate products in:
    1. National Income and Product Accounting
    2. Input-output method
    3. Flow-of Funds system
    4. Federal reserve Index of Industrial Production.” (Don’t panic if you can’t do (d), but if you can you’ll get a premium.
      Hint: there is more in this question, and particularly in part (a) than meets the eye. Consider the whole rationale of the methods.
  3. Write a comprehensive essay on the subject of “The Rationale of Investment Decisions.” Consider as many cases as you can, but in each case specify clearly the assumptions made. (Don’t forget to include an undeveloped country case.) Can you generalize?
  4. Write a comprehensive and critical essay on the subject of “Price Flexibility and Employment.” Survey the relevant literature beginning with Keynes’ General Theory, and indicate clearly the nature of the assumptions, the definition of the concepts (hint: money), and the essence of the conclusions. What practical recommendations follow from your discussion?

 

Source: Duke University, David M. Rubenstein Library. Economists’ Papers Archives. Papers of Evsey D. Domar, Box 16, Folder “Macroeconomics, Final Exams”.

Image Source: Evsey D. Domar photo at the M.I.T. Museum website.

Categories
Austria Economists Exam Questions Johns Hopkins Methodology

Johns Hopkins. Final exam for Fritz Machlup’s methodology course, 1956

 

 

Besides the questions for the final exam in Fritz Machlup’s course on the methodology of economics from the first semester of the 1955-56 academic year at Johns Hopkins University, I include the following photo from the 1956 yearbook Hullabaloo (p. 15) that identifies neither the speaker nor the seminar. While this is about as generic a seminar photo as one can imagine, I have something more than a mere suspicion that we are looking at Fritz Machlup in action. Perhaps some visitor with a keener forensic eye can confirm or reject my tentative identification in a comparison of the above portrait of Machlup reading himself with the speaker in the mystery seminar. The third man on the right, counting from the speaker, sure looks like a young Evsey Domar.

My hunch is based on the following picture of almost certainly the same seminar room in 1963 from the Carl Christ memorial website at Johns Hopkins.

 

______________

THE JOHNS HOPKINS UNIVERSITY
METHODOLOGY
18.601

Professor Fritz Machlup
January 27, 1956

Answer five questions, one from each group.
Write on loose sheets of paper; start a new sheet for each question.
Identify each sheet by the Question Number in the left corner and your Examination Number (which you draw before the examination) in the right corner; your name should appear nowhere.
You are on your honor not to use notes or to give or accept advice.

I.

  1. According to Poincaré, “a priori propositions are irrefutable because they are really firm resolutions to carry on the scientific game according to certain rules or stipulations.” Nevertheless, Morris Cohen considers a priori principles as “methodologic or regulative principles which enable us to organize our factual knowledge” and as “expressive of the fundamental nature of things,” What light does this view throw upon the methodological discussions of Hutchison, Kaufmann, Mises, and Knight?
  2. “While the deductive method might be applicable to a simple and stationary condition of industry, it becomes valueless in face of the increasing complexity of the modern economic world.” What was John Neville Keynes’ reaction to this point of view?
  3. “Just as the same proposition may express both a universal and a historical, or both a verbal and a real judgment, so it may express both a positive and a normative judgment.” (Fraser, Economic Thought and Language). First explain each of the three sets of antonyms and then explain and illustrate the statement.

II.

  1. Contrast and compare the logical nature of introspectionism and sensationalism as expounded by Felix Kaufmann or Morris Cohen.
  2. Give a reasoned explanation of Kaufmann’s distinction between three meanings of probability, one “relating to empirical knowledge as such”, another relating “to synthetic propositions undecided in a given scientific situation,” and a third referring to “the relative frequency of an attribute” within a certain collective.
  3. Felix Kaufmann, having made a distinction between empirical laws and theoretical laws, states: “Whereas we have both types of laws in natural science, there are, as I see it, no empirical laws established in social science, and even the tendency to establish such laws is not very strong. But if we consider the significance of theoretical laws in natural science, we cannot regard this as constituting a fundamental difference between the methods of natural science and those of social science.” Explain and discuss this statement in a way intelligible to someone who has not read Kaufmann’s writings.

III.

  1. Explain what Ludwig von Mises means by ”methodological apriorism”, “methodological individualism”, and “methodological singularism”.
  2. “In the history of applied Economics, the work of a Jevons, a Menger, a Bowley, has much more claim on our attention than the work of, say, a Schmoller, a Veblen, or a Hamilton.” What is Robbins driving at with this [last word cut off, “statement?” matches the spacing of the tips to the letter “t” that are still visible]
  3. Hutchison implies that pure theory may help the analyst to formulate questions to be answered by empirical studies: “The constant object of the scientist…is to compel the facts of experience to answer his questions definitely ‘yes’ or ‘no’…” Robbins appears to reverse the relationship: “Realistic studies may suggest the problem to be solved….But it is theory and theory alone which is capable of supplying the solution.” Discuss the paradox from the point of view of any of the other writers on methodology.

IV.

  1. If the description or institutional part of economics is viewed by Professor Knight as lying in the domain of cultural anthropology rather than economics proper, does this mean that in institutional inquiries sense observation assumes greater emphasis than intercommunication and interpretation? If not, why does Knight distinguish institutional from theoretical economics?
  2. “There are no better terms available to describe the difference between the approach of the natural and the social sciences than to call the former objective and the latter subjective.” (Hayek, “Scientism and the Study of Society”.) Explain the meaning of the essential terms employed and the statement as a whole.
  3. Hayek said: “It is only in so far as some sort of order arises as a result of individual action but without being designed by any individual that a problem is raised which demands a theoretical explanation.” Explain.

V.

  1. “Economics is in fact the only science which enjoys the advantage of an automatic quantification of its subject matter.” (Parsons, “Sociological Elements in Economic Thought”). Explain and discuss.
  2. Parsons distinguishes the following ideal types of criticism of abstract economic theory: (1) supplementary positivistic empiricism: (2) radical positivistic empiricism; (3) romantic empiricism; (4) supplementary non-economic sociology. Characterize each in a brief statement illustrated by examples.
  3. Discuss Veblen’s principal categories of human action—especially the “pragmatic” versus the “workmanlike” type—and compare them with the general “rational” type and the narrower “economic” type used in the abstract theories of traditional economics.
  4. On what grounds do Professors Herskovits and Knight reject and defend, respectively, the concept of the “economic man” as a useful tool of economic analysis?

 

Source:  Johns Hopkins University. The Ferdinand Hamburger, Jr. Archives. Department of Political Economy, Series 6. Box 3/1. Folder: “Department of Political Economy, Graduate Exams, 1933-1965”.

Image Source:  Johns Hopkins University yearbook, Hullabaloo, 1957, p. 28.